Libro de Desigualdades
Libro de Desigualdades
Libro de Desigualdades
i
ii
INTRODUCCIÓN
iii
iv
de las pruebas de las olimpiadas internacionales de matemática (IMO, por sus siglas
en inglés) desde el año 59 al año 2008. En algunos casos, los problemas están acom-
pañados de más de una solución de modo que el lector conozca una mayor diversidad
de estrategias para encarar los ejercicios.
Para la elaboración del texto he utilizado como fuente libros editados en Estados
Unidos, Europa y Asia. Asimismo, he recurrido a valiosa información de Internet y he
sistematizado la experiencia adquirida en la enseñanza de estudiantes que participan
en olimpiadas matemáticas.
Es recomendable, para abordar el texto, que el lector tenga conocimientos básicos
de desigualdades, de manera que pueda profundizar con mayor facilidad en los temas
propuestos. Es importante que estudie con detenimiento la teorı́a, revise las aplica-
ciones y luego intente resolver los problemas antes de recurrir a la solución.
Quiero hacer público mi agradecimiento al Instituto de Ciencias y Humanidades
(ICH) por su apoyo en la investigación y edición del texto. A Jorge Tipe Villanueva,
por sus importantes sugerencias, a Juan Neyra Faustino, por su colaboración en el
digitado en LATEXdel material y, en especial, a mi esposa Magda, a mi hijo Taylor y
a mis padres Francisco y Serafina por su constante y siempre desinteresado apoyo.
A todos ellos, mi profundo agradecimiento.
1. Números Reales 1
1.1. Cuerpos . . . . . . . . . . . . . . . . . . . . . . . . . . . . . . . . . . 1
1.1.1. Axiomas de adición. . . . . . . . . . . . . . . . . . . . . . . . 1
1.1.2. Axiomas de multiplicación. . . . . . . . . . . . . . . . . . . . . 2
1.2. Cuerpos Ordenados . . . . . . . . . . . . . . . . . . . . . . . . . . . . 3
1.3. Aplicaciones . . . . . . . . . . . . . . . . . . . . . . . . . . . . . . . . 13
3. Desigualdad de Cauchy-Schwarz 37
3.1. El Lema de Titu . . . . . . . . . . . . . . . . . . . . . . . . . . . . . 39
3.2. Desigualdad de Schür . . . . . . . . . . . . . . . . . . . . . . . . . . . 41
3.3. Dos sustituciones muy útiles . . . . . . . . . . . . . . . . . . . . . . . 41
3.4. Aplicaciones . . . . . . . . . . . . . . . . . . . . . . . . . . . . . . . . 43
6. Desigualdad de Reordenamientos 79
v
vi ÍNDICE GENERAL
8. Espacio Métrico 97
8.1. Desigualdad de Hölder . . . . . . . . . . . . . . . . . . . . . . . . . . 102
8.2. Desigualdad de Minkonski . . . . . . . . . . . . . . . . . . . . . . . . 103
11.Problemas 129
12.Soluciones 149
Bibliografı́a 275
CAPÍTULO 1
NÚMEROS REALES
1.1. Cuerpos
Un cuerpo es un conjunto K con dos operaciones llamadas adición y multipli-
cación, que satisfacen ciertas condiciones llamadas axiomas de cuerpo.
x + y ∈ K, ∀ x, y ∈ K.
A2 . Conmutatividad.
x + y = y + x, ∀ x, y ∈ K.
A3 . Asociatividad.
(x + y) + z = x + (y + z), ∀ x, y, z ∈ K.
A4 . Elemento neutro.
Existe 0 ∈ K tal que
x + 0 = x, ∀ x ∈ K.
1
2 1. Números Reales
A5 . Simétrico.
Para todo x ∈ K existe (−x) ∈ K tal que
x + (−x) = 0.
x · y ∈ K, ∀ x, y ∈ K.
M2 . Conmutatividad.
x · y = y · x, ∀ x, y ∈ K.
M3 . Asociatividad.
M4 . Elemento neutro.
Existe 1 ∈ K tal que 1 6= 0 y
x · 1 = x, ∀ x ∈ K.
M5 . Simétrico.
Para todo x 6= 0 ∈ K existe el inverso de x denotado por x−1 ∈ K tal que
x · x−1 = 1.
1.2. Cuerpos Ordenados 3
D1 . Axioma de distributividad.
x · (y + z) = x · y + x · z, ∀ x, y, z ∈ K.
Ejemplos de cuerpos
m
1. Q = { | m ∈ Z; n ∈ Z+ }.
n
2. R = Q ∪ Q′ ; Q′ : conjunto de los irracionales.
P1 . x + y ∈ P y x · y ∈ P, ∀ x, y ∈ P.
x = 0 ∨ x ∈ P ∨ −x ∈ P.
K = P ∪ (−P) ∪ {0}.
a2 = a · a ∈ P;
en el segundo caso
a2 = (−a) · (−a) ∈ P.
Prueba. En efecto
x < y ⇔ y − x > 0;
y < z ⇔ z − y > 0;
(y − x) + (z − y) ∈ P
(z − x) ∈ P
z−x>0
x < z.
2
Teorema 1.2 (Tricotomı́a)
Dados x, y ∈ K, sólo se cumple una de las tres condiciones
x = y ∨ x < y ∨ x > y.
1.2. Cuerpos Ordenados 5
Prueba. En efecto
∀ x, y ∈ K, y − x = 0 ∨ y − x ∈ P ∨ y − x ∈ −P,
equivale a
y = x ∨ x < y ∨ −(y − x) ∈ P
x−y ∈P
y<x
x>y
∴ y = x ∨ x < y ∨ x > y.
2
Teorema 1.3 (Monotonicidad de la adición)
x < y ⇔ x + z < y + z, ∀ z ∈ K.
Prueba. En efecto
x<y
⇔ y−x> 0
⇔ (y + z) − (x + z) > 0
⇔ x + z < y + z.
2
Teorema 1.4 (Monotonicidad de la multiplicación)
Prueba. En efecto
x<y ∧ z>0
⇔ y−x>0 ∧ z >0
⇔ (y − x); z ∈ P
6 1. Números Reales
entonces
(y − x) · z ∈ P
yz − xz ∈ P
yz − xz > 0
xz < yz.
Análogamente
(y − x) · (−z) ∈ P
−yz + xz ∈ P
xz − yz > 0
xz > yz.
2
Teorema 1.5
x < y ∧ a < b ⇒ x + a < y + b.
Prueba. En efecto
Teorema 1.7
0 < x < y ∧ 0 < a < b ⇒ xa < yb.
Prueba. En efecto
Teorema 1.8
1
x>0 ⇔ > 0.
x
Prueba. En efecto
Prueba. En efecto
2
Teorema 1.10
xy < 0 ⇔ (x < 0 ∧ y > 0) ∨ (x > 0 ∧ y < 0).
8 1. Números Reales
(⇒) a < b.
Como ab > 0 entonces tenemos dos casos:
1 1
i. Si a > 0 ∧ b > 0 entonces >0 ∧ > 0, luego
a b
1 1 b−a
− = > 0, pues a < b
a b ab
1 1
⇒ − >0
a b
1 1
⇒ > .
a b
ii. Para a < 0 ∧ b < 0 el procedimiento es análogo.
1 1 1 1
(⇐) Como > entonces ab > ab, efectuando tenemos b > a, de donde
a b a b
a < b.
2
Teorema 1.12
En un cuerpo ordenado K, las siguientes afirmaciones son equivalentes:
(i) x ≤ x0 , ∀ x ∈ X.
(i) ∀ y ∈ Y ; y ≥ y0 .
Ejemplo 1.2
Sean A = [−3, 5], B = h−3, 5i, entonces
Prueba. Supongan que sup (B) no es 5; sea sup (B) = x0 , significa que x0 < 5:
x0 + 5 x0 + 5
Como x0 < < 5, entonces m = ∈ B, de donde
2 2
x0 < m; (1.1)
pero x ≤ x0 , ∀x ∈ B; en particular haciendo x = m, tenemos
m ≤ x0 ; (1.2)
de (1.1) y (1.2), tenemos
m ≤ x0 < m
∴ sup (B) = 5.
(a − b)2 ≥ 0
⇔ a2 − 2ab + b2 ≥ 0
⇔ a2 + b2 ≥ 2ab.
(ii) De (i)
a2 + b2 ≥ 2ab
⇔ a2 + 2ab + b2 ≥ 4ab
⇔ (a + b)2 ≥ 4ab
⇔ 4ab ≤ (a + b)2 .
1
(−y) + ≥ 2;
(−y)
1
−y − ≥ 2; multiplicando por (−1)
y
1
y+ ≤ −2.
y
1.3. Aplicaciones
1. Sean a, b números reales no nulos, determine el mı́nimo valor de
a8 a4 a2 b8 b4 b2
+ + + + + .
b8 b4 b2 a8 a4 a2
Solución.
a8 a4 a2 b8 b4 b2 a8 b8 a4 b4 a2 b2
+ + + + + = + + + + +
b8 b4 b2 a8 a4 a2 b8 a8 b4 a4 b2 a2
≥ 2 + 2 + 2 = 6.
ax + by ≥ ay + bx.
(a − b)x ≥ (a − b)y
⇔ ax − bx ≥ ay − by
⇔ ax + by ≥ ay + bx.
a2 + b2 + c2 ≥ ab + ac + bc.
14 1. Números Reales
a2 + b2 ≥ 2ab; similarmente
b2 + c2 ≥ 2bc;
c2 + a2 ≥ 2ca.
Prueba. √ √ √
a, b, c ∈ R+ ⇒ a, b, b ∈ R+ ,
luego
√ √
( a − b)2 ≥ 0
√
⇔ a − 2 ab + b ≥ 0
√
⇔ a + b ≥ 2 ab, similarmente
√
b + c ≥ 2 bc,
√
c + a ≥ 2 ca,
a2 b2
+ ≥ a + b.
b a
a3 + b3
≥a+b
ab
⇔ a3 + b3 ≥ ab(a + b)
⇔ (a + b)(a2 − ab + b2 ) ≥ ab(a + b); a + b > 0
⇔ a2 − ab + b2 ≥ ab
⇔ a2 − 2ab + b2 ≥ 0
⇔ (a − b)2 ≥ 0.
a2 + b2 b2 + c2 c2 + a2
+ + ≥ 2(a + b + c).
c a b
a2 b2
+ ≥ a + b; similarmente
b a
b2 c2
+ ≥ b + c;
c b
c2 a2
+ ≥ a + c;
a c
a2 + b2 b2 + c2 c2 + a2
+ + ≥ 2(a + b + c).
c a b
a2 b2 c2
+ + ≥ 1.
a2 + ab + b2 b2 + bc + c2 c2 + ca + a2
a2 b2 c2
+ + ≥1
m n p
a2 b2 c2
⇔ + + −1 ≥0
m n p
1 1 1 a2 b2 c2
⇔ ( + + )( + + − 1) ≥ 0
m n p m n p
2 2 2 2
a b c b c2 a2 c2 a2 b2 1 1 1
⇔ 2
+ 2
+ 2
+ + + + + + − − − ≥0
m n p mn mp mn np mp np m n p
2 2 2
a b c (a2 + b2 ) (a2 + c2 ) (b2 + c2 ) 1 1 1
⇔ + + + + + − − − ≥0
m2 n2 p2 mn mp np m n p
a2 b2 c2 ab (a2 + ab + b2 ) ac (a2 + ac + c2 ) bc
⇔ 2
+ 2
+ 2
− + − + −
m n p mn mn mp mp np
2 2
(b + bc + c ) 1 1 1
+ − − − ≥0
np m n p
a2 b2 c2 ab ac bc m p n 1 1 1
⇔ 2
+ 2
+ 2
− − − + + + − − − ≥0
m n p mn np np mn mp np m n p
a 2 b 2 c 2 a b a c b c
⇔ + + ≥ + + ,
m n p m n m p n p
(1 + x2 )(1 + y 2) ≥ (1 + xy)2 .
x2 + y 2 ≥ 2xy, ∀ x, y ∈ R
⇔ 1 + x2 + y 2 + x2 y 2 ≥ 1 + 2xy + x2 y 2
⇔ (1 + x2 )(1 + y 2 ) ≥ (1 + xy)2 .
a2 y 2 − 2abxy + b2 x2 ≥ 0
⇔ a2 y 2 + b2 x2 ≥ 2abxy
⇔ a2 x2 + b2 y 2 + a2 y 2 + b2 x2 ≥ a2 x2 + b2 y 2 + 2abxy
⇔ a2 (x2 + y 2) + b2 (y 2 + x2 ) ≥ (ax)2 + (by)2 + 2(ax)(by)
⇔ (a2 + b2 )(x2 + y 2) ≥ (ax + by)2
x2 + y 2 + 2
11. Sea f (x, y) = p , donde x, y son números reales, pruebe que
(1 + x2 )(1 + y 2 )
el menor valor de f es 2.
p √
x2 + y 2 + 1 > x y 2 + 1 + y x2 + 1,
p
Prueba. Como en el segundo miembro tenemos x y 2 + 1, entonces pode-
mos a partir de:
p
(x − y 2 + 1)2 ≥ 0
p
⇔ x2 + y 2 + 1 − 2x 1 + y 2 ≥ 0
p
⇔ x2 + y 2 + 1 ≥ 2x 1 + y 2; simirlamente
√
x2 + y 2 + 1 ≥ 2y 1 + x2 .
(
x2 = 1 + y 2
⇒
y 2 = 1 + x2
⇒ x2 + y 2 = x2 + y 2 + 2,
p √
∴ x2 + y 2 + 1 > x y 2 + 1 + y x2 + 1.
2(1 − a + a2 )(1 − b + b2 ) ≥ 1 + a2 b2 ,
14. Resuelva
15. (Rusia 1995). Pruebe que para todo x, y números reales positivos
x y 1
+ ≤ .
x4 + y 2 x2 + y 4 xy
Prueba. Como
1 1
x4 + y 2 ≥ 2x2 y ⇔ ≤ 2
+yx4
2 2x y
x 1
⇔ 4 2
≤
x +y 2xy
similarmente
y 1
≤
x2 +y 4 2xy
x y 1
+ 2 ≤ .
x4 +y 2 x +y 4 xy
Prueba. Como
a2 + b2 ≥ 2ab
⇔ a2 − ab + b2 ≥ ab
⇔ 4a2 − 4ab + 4b2 ≥ 4ab
⇔ 8a2 − 4ab + 5b2 ≥ 4a2 + 4ab + b2
⇔ 8a2 − 4ab + 5b2 ≥ (2a + b)2
8a2 − 4ab + 5b2
⇔ ≥ 2a + b; similarmente
2a + b
8b2 − 4bc + 5c2
≥ 2b + c;
2b + c
8c2 − 4ac + 5a2
≥ 2c + a;
2c + a
S ≥ 3(a + b + c).
1 1 2 a2 + b2
2 2
+ 2 2
≤
a + ab + b a − ab + b 3 a2 b2
1 1 2(a2 + b2 )
+ =
a2 + ab + b2 a2 − ab + b2 a4 + a2 b2 + b4
acotando el denominador
1.3. Aplicaciones 23
a4 + b4 ≥ 2a2 b2
⇔ a4 + a2 b2 + b4 ≥ 3a2 b2
1 1
⇔ 4 2 2 4
≤ 2 2
a +a b +b 3a b
2 2
2(a + b ) 2(a2 + b2 )
⇔ 4 ≤ .
a + a2 b2 + b4 3a2 b2
x2 + y 2 + z 2
zx + yz ≤ √ ,
2
x2 + y 2 + z 2
∴ zx + yz ≤ √ .
2
z
La igualdad ocurre si y sólo si x = y = √ . 2
2
24 1. Números Reales
2(a3 + b3 + c3 ) ≥ a2 (b + c) + b2 (a + c) + c2 (a + b),
2(a3 + b3 + c3 ) ≥ a2 (b + c) + b2 (a + c) + c2 (a + b).
Ejemplo 2.1√ √ √ √ √ √
|2| = 2; | − 3| = −(− 3) = 3; |1 − 2| = −(1 − 2) = 2 − 1.
Teorema 2.1
Prueba.
x;
para x > 0;
máx {x; −x} = 0; para x = 0; = |x|.
−x; para x < 0.
2
25
26 2. Valor Absoluto y Máximo Entero
Teorema 2.2
Para elementos arbitrarios x, y reales, se cumplen
(i) | − x| = |x|.
x |x|
(v) | | = .
y |y|
Prueba.
x;
x > 0;
|x| = 0; x = 0;
−x; x < 0;
entonces
2
x ;
x < 0;
|x|2 = 02 ; x = 0;
2
(−x) ; x > 0;
= x2 , ∀ x ∈ R.
2.1. Valor Absoluto 27
(iii)
|x|2 = x2
p √
|x|2 = x2
√
|x| = x2 .
(iv)
(x · y)2 = x2 · y 2
p p
(x · y)2 = x2 · y 2
√ p
|xy| = x2 · y 2
|xy| = |x| · |y|
Teorema 2.3
Para cualesquiera x, y, z números reales, se cumplen
(i) |x| ≥ x.
(v) |x − z| ≤ |x − y| + |y − z|.
Prueba.
(iii)
|xy| ≥ xy ⇔ 2|x||y| ≥ 2xy
⇔ x2 + y 2 + 2|x||y| ≥ x2 + y 2 + 2xy
⇔ |x|2 + |y|2 + 2|x||y| ≥ (x + y)2
⇔ (|x| + |y|)2 ≥ (x + y)2
⇔ |x| + |y| ≥ |x + y|
⇔ |x + y| ≤ |x| + |y|.
(iv)
Similarmente
De (2.1) y (2.2)
(v)
|x − z| = |(x − y) + (y − z)| ≤ |x − y| + |y − z|
|x − z| ≤ |x − y| + |y − z|.
2
2.2. Máximo Entero o Parte Entera 29
Ejemplo 2.2
Teorema 2.4
Para todo x número real:
Prueba.
(i) De la definición
[x] = n ⇔ n ≤ x < n + 1; n ∈ Z
0 ≤ x − [x] < 1.
30 2. Valor Absoluto y Máximo Entero
∴ [[x]] = [x].
2
Teorema 2.5
Para todo x número real y n número entero, se cumple que
(i) [x + n] = [x] + n.
(
−[x]; si x = [x];
(ii) [−x] =
−[x] − 1; si x 6= [x].
h x i [x]
(iii) = , n ≥ 1.
n n
Prueba.
(i) Sea m = [x + n]. De la definición
[x + n] = m ⇔ m ≤ x + n < m + 1
⇔ (m − n) ≤ x < (m − n) + 1; (m − n) ∈ Z
⇔ [x] = m − n
⇔ [x] + n = m
⇔ [x] + n = [x + n]
También [x] = [k + p] = k
h
[x] k nq + r ri
= = = q+ = q,
n n n n
hxi
[x]
de donde = .
n n
2
{x} = x − [x].
De la definición se tiene
n−1
X
k
x+ = [nx].
k=0
n
1 1 2 3 1 n−1 1
f x+ = x+ + x+ + x+ +··· x+ + − n(x + )
n n n n n n n
1 1 2 n−1
f x+ = x+ + x+ +···+ x + + [x + 1] − [nx + 1]
n n n n | {z }
[x]+1−([nx]+1)
1 2 n−1
= [x] + x + + x+ +···+ x + − [nx]
n n n
= f (x),
de donde f (x) = 0, ∀ x ∈ R.
n−1
X
k
∴ x+ = [nx].
k=0
n
2
Teorema 2.7
Sea a un número entero y b un número entero positivo, entonces
hai b − a − 1
+ = 0.
b b
2.3. Aplicaciones
x2
1. Resuelva = 1.
16
Solución.
x2 x2
=1 ⇔ 1≤ <2
16 16
⇔ 16 ≤ x2 < 32
√ √
⇔ 4 ≤ x < 4 2 ∨ 4 2 < x ≤ −4.
D h √ E
√ i
∴ C.S. = −4 2, −4 ∪ 4, 4 2 .
2
2.3. Aplicaciones 33
2. Resuelva la inecuación
2 √
x − 1 > 2.
Solución. La inecuación:
√
[x2 − 1] > 2 ⇔ [x2 − 1] ≥ 2
⇔ x2 − 1 ≥ 2
⇔ x2 ≥ 3
√ √
⇔ x≥ 3 ∨ x≤− 3
D
√ i h√ E
∴ C.S. = −∞, 3 ∪ 3, +∞ .
3. Resuelva en Z, la ecuación
5−x
2 =− x−2 .
2 4
Luego resolveremos
5−x x−2
=−
4 4
5−x x−2
⇔ + = 0.
4 4
34 2. Valor Absoluto y Máximo Entero
4. Pruebe que
√ √ √
[ n + n + 1] = [ 4n + 2]
√ √ √
Prueba. Basta probar que entre n+ n+1 y 4n + 2 no existe un
entero.
La prueba la haremos por contradicción; en efecto, supongamos que existe
r ∈ Z+ tal que
√ √ √
n + n + 1 < r < 4n + 2
√ √ √
⇔ ( n + n + 1)2 < r 2 < ( 4n + 2)2
p
⇔ 2n + 1 + 2 n(n + 1) < r 2 < 4n + 2
pero
√
n< n2 + n
√
⇔ 2n < 2 n2 + n
√
⇔ 2n + 2n + 1 < 2 n2 + n + 2n + 1,
√
4n + 1 < 2 n2 + n + 2n + 1 < r 2 < 4n + 2,
2.3. Aplicaciones 35
√ √ √
∴ [ n + n + 1] = [ 4n + 2].
2
es independiente de n.
41 − n 41 − n
n − 12 + 25 n − 12 + 25
= ; pues n − 12 ∈ Z
3 3
24n − 259 24n − 259 24n − 259
25 25 3
=
=
=
3 3 25
24n − 259 −259
3 8n + 3 8n − 87
=
25 = 25 = 25
8n + 13 − 100 8n + 13 8n + 13
= = −4 = − 4,
25 25 25
n − 17
n − 12 −
8n + 13 25
luego −4 =
.
25 3
n − 17
n − 12 −
8n + 13 25
∴ −
= 4.
25 3
2
CAPÍTULO 3
DESIGUALDAD DE
CAUCHY-SCHWARZ
37
38 3. Desigualdad de Cauchy-Schwarz
donde
∆ = (−2B)2 − 4AC ≤ 0
4B 2 − 4AC ≤ 0
B 2 ≤ AC
n
! n
! n
!2
X X X
Segunda Prueba. Haciendo M = a2i b2i ; N = ai bi , en-
i=1 i=1 i=1
tonces tenemos que
3.1. El Lema de Titu 39
n
! n
! n
!2
X X X
M −N = a2i b2i − ai bi
i=1 i=1 i=1
n
X X n
X X
= a2i b2i + a2i b2j − a2i b2i − 2 ai bj aj bi
i=1 i6=j i=1 1≤i<j≤n
X X
= a2i b2j − 2 ai bj aj bi
i6=j 1≤i<j≤n
X X
= a2i b2j + a2j b2i − 2 ai bj aj bi
1≤i<j≤n 1≤i<j≤n
X
= a2i b2j − 2ai bj aj bi + a2j b2i
1≤i<j≤n
X
= (ai bj − aj bi )2 ≥ 0.
1≤i<j≤n
Entonces
M −N ≥ 0
M ≥ N. [9]
2
s s 2
a21 a22 a2n a21 √ a2n √
+ +···+ (x1 + x2 + · · · + xn ) ≥ · x1 + · · · + · xn
x1 x2 xn x1 xn
a21 a22 a2
+ +···+ n (x1 + x2 + · · · + xn ) ≥ (a1 + a2 + · · · + an )2 .
x1 x2 xn
40 3. Desigualdad de Cauchy-Schwarz
a1 a2
y la igualdad se tiene si y sólo si = .
x1 x2
Aplicando el resultado dos veces se tiene que
a21 a22 a2n a2n+1 (a1 + a2 + · · · + an )2 a2n+1
+ +···+ + ≥ +
x1 x2 xn xn+1 x1 + x2 + · · · + xn xn+1
(a1 + a2 + · · · + an+1 )2
≥ . [11]
x1 + x2 + · · · + xn+1
2
3.2. Desigualdad de Schür 41
x − z ≥ y − z, x − y ≥ 0
(x − y)(x − z) ≥ (x − y)(y − z), xn ≥ y n
xn (x − y)(x − z) ≥ y n (x − y)(y − z). (3.1)
También
(z − x)(z − y) ≥ 0, pues x ≥ z, y ≥ z
n
⇔ z (z − x)(z − y) ≥ 0. (3.2)
xyz = x + y + z + 2
tenemos
xyz + xy + xz + yz + x + y + z + 1 = xy + xz + yz + 2(x + y + z) + 3
(x + 1)(y + 1)(z + 1) = (xy + x + y + 1) + (xz + x + z + 1)
+(yz + y + z + 1)
(x + 1)(y + 1)(z + 1) = (x + 1)(y + 1) + (x + 1)(z + 1)
+(y + 1)(z + 1)
1 1 1
⇒ 1= + + .
z+1 y+1 x+1
1 1 1
Haciendo a = , b= , c= , se tiene a + b + c = 1 y
x+1 y+1 z+1
1−a b+c c+a a+b
x= = , y= , z= .
a a b c
a b c
x= , y= , z= ,
b+c c+a a+b
1 1 1 1
= + + + 2,
xyz x y z
3.4. Aplicaciones
1. Sean a, b números reales tales que a2 + b2 = 1; halle la variación de 2a + b.
√ √
∴ 2a + b ∈ [− 5, 5].
2 !
√ 1 √ 1 √ √ 1 2 1 2
2 2
x−1· + y−1· ≤ x−1 + y−1 +
x y x y
√ √ 2
x−1 y−1 1 1
⇔ + ≤ (x − 1 + y − 1) +
x y x2 y 2
√ √ 2
x−1 y−1 1 1
⇔ + ≤ (x + y − 2) + .
x y x2 y 2
3. Pruebe que
Luego tenemos
|a + 3b + 9c|
f (a, b, c) = √ .
a2 + b2 + c2
√
como f (a, b, c) ≥ 0 entonces 0 ≤ f (a, b, c) ≤ 91.
√
∴ El máximo valor de f es 91, y esto ocurre si y sólo si (a, b, c) = (1, 3, 9).
2
Solución. Sean los pares (a, b), (sen α, cos α). Utilizando Cauchy-Schwarz
tenemos:
√ √
∴ − a2 + b2 ≤ f (α) ≤ a2 + b2 .
∴ a2 + b2 + c2 ≥ 14.
aplicando Cauchy-Schwarz:
8. Pruebe que
x2 y2 z2
x+y+z ≤2 + + ,
y+z x+z x+y
Prueba. Como
2
2 x √ y √ z √
(x + y + z) = √ · y+z+ √ · x+z+ √ · x+y
y+z x+z x+y
2
x y2 z2
≤ + + (y + z + x + z + x + y)
y+z x+z x+y
2
2 x y2 z2
(x + y + z) ≤ 2 (x + y + z) + + ;
y+z x+z x+y
cancelando x + y + z > 0
x2 y2 z2
x+y+z ≤2 + + .
y+z x+z x+y
1 1 1
9. Sean x, y, z > 1 tales que + + = 2, pruebe que
x y z
√ √ p √
x+y+z ≥ x−1+ y − 1 + z − 1.
√ √ √
x−1 y−1 z−1 √ √ √
√ , √ , √ y x, y, z ;
x y z
√ √ √ 2
x−1 √y−1 √ z−1 √
√ · x+
√ · y+ √ · z
x y z
x−1 y−1 z−1
≤ + + · (x + y + z)
x y z
48 3. Desigualdad de Cauchy-Schwarz
√ 2
p √ 1 1 1
⇔ x−1+ y−1+ z−1 ≤ 1− +1− +1−
x y z
· (x + y + z)
1 1 1
= 3− + + (x + y + z)
x y z
= (3 − 2) (x + y + z)
= (x + y + z).
√ p √ 2
Entonces x−1+ y−1+ z−1 ≤ x + y + z.
√ p √ √
∴ x − 1 + y − 1 + z − 1 ≤ x + y + z.
3
La igualdad ocurre si y sólo si x = y = z = . 2
2
a21 a22 a2 a2
+ + · · · + n−1 + n ≥ a1 + a2 + · · · + an .
a2 a3 an a1
a b c 3
+ + ≥ ,
b+c a+c a+b 2
Prueba.
a b c a2 b2 c2
+ + = + +
b+c a+c a+b ab + ac ba + bc ca + cb
(a + b + c)2
≥
(ab + ac) + (ba + bc) + (ca + cb)
(a + b + c)2
= ,
2(ab + bc + ca)
(a + b + c)2 3
≥ .
2(ab + bc + ca) 2
Como
a2 + b2 + c2 ≥ ab + ac + bc
⇔ a2 + b2 + c2 + 2(ab + ac + bc) ≥ 3(ab + ac + bc)
(a + b + c)2 3
⇔ ≥ .
2(ab + bc + ca) 2
a3 b3 c3 a+b+c
2 2
+ 2 2
+ 2 2
≥ .
a + ab + b b + bc + c c + ca + a 3
a4 b4 c4
+ + ,
a3 + a2 b + ab2 b3 + b2 c + bc2 c3 + c2 a + ca2
50 3. Desigualdad de Cauchy-Schwarz
a4 b4 c4
+ +
a3 + a2 b + ab2 b3 + b2 c + bc2 c3 + c2 a + ca2
(a2 + b2 + c2 )2
≥ 3
a + b3 + c3 + ab(a + b) + bc(b + c) + ca(c + a)
(a2 + b2 + c2 )2
=
(a + b + c)(a2 + b2 + c2 )
a2 + b2 + c2
= ,
a+b+c
entonces
a3 b3 c3 a2 + b2 + c2
+ + ≥ .
a2 + ab + b2 b2 + bc + c2 c2 + ca + a2 a+b+c
Pero
a2 + b2 + c2 ≥ ab + ac + bc
⇔ 2(a2 + b2 + c2 ) ≥ 2(ab + ac + bc)
⇔ 3(a2 + b2 + c2 ) ≥ a2 + b2 + c2 + 2(ab + ac + bc)
⇔ 3(a2 + b2 + c2 ) ≥ (a + b + c)2
a2 + b2 + c2 a+b+c
⇔ ≥ .
a+b+c 3
De donde
a3 b3 c3 a+b+c
2 2
+ 2 2
+ 2 2
≥ .
a + ab + b b + bc + c c + ca + a 3
a b c 3
+ + ≥ ,
b+c a+c a+b 2
a b c
Prueba. Haciendo x = , y= , z= .
b+c c+a a+b
Entonces es suficiente probar que si x, y, z > 0 con xy + yz + zx + 2xyz = 1,
entonces x + y + z ≥ 3/2.
3
Supongamos que x + y + z < .
2
Como
(x + y + z)2 3
xy + yz + zx ≤ ⇒ xy + yz + zx <
3 4
3
x+y+z 1
xyz ≤ ⇒ 2xyz <
3 4
de donde
3 1
1 = (xy + yz + zx) + 2xyz < + = 1.
4 4
52 3. Desigualdad de Cauchy-Schwarz
x + y + z ≥ 3/2.
15. Pruebe que si x, y, z son números reales positivos y xyz = x+y +z +2 entonces
xy + yz + zx ≥ 2(x + y + z).
b+c c+a c+a a+b a+b b+c
+ +
a b b c c a
b+c c+a a+b
≥2 + +
a b c
⇔ a3 + b3 + c3 + 3abc ≥ ab(a + b) + bc(b + c) + ca(c + a)
⇔ a3 + b3 + c3 + 3abc ≥ a2 (b + c) + b2 (a + c) + c2 (a + b)
⇔ (a3 − a2 (b + c) + abc) + (b3 − b2 (a + c) + abc) + (c3 − c2 (a + b) + abc) ≥ 0
⇔ a(a − b)(a − c) + b(b − a)(b − c) + c(c − a)(c − b) ≥ 0.
2 2 2
b+c c+a a+b
−1 −1 −1
a b c 3
2 + 2 + 2 ≥ .
b+c c+a a+b 5
+1 +1 +1
a b c
Utilizando la sustitución
con xyz = x + y + z + 2, x + y + z ≥ 6.
Aplicando el Lema de Titu
(x + y + z − 3)2 3
≥ .
x2 + y 2 + z 2 + 3 5
⇔ 5(x + y + z − 3)2 ≥ 3(x2 + y 2 + z 2 ) + 9 (3.3)
Haciendo
Reemplazando en (3.3)
efectuando
Pero de la aplicación 15
xy + yz + zx ≤ 2(x + y + z) = 2s
⇔ xy + yz + zx ≤ 2s
⇔ 3(xy + yz + zx) ≤ 6s
⇔ −6s ≤ −3(xy + yz + zx). (3.5)
s2 − 15s + 18 ≥ −6s
⇔ s2 − 9s + 18 ≥ 0
⇔ (s − 3)(s − 6) ≥ 0,
es verdadero, pues s ≥ 6.
La igualdad ocurre si y sólo si a = b = c. 2
CAPÍTULO 4
DESIGUALDAD DE LA MEDIA
ARITMÉTICA - MEDIA GEOMÉTRICA
x1 + x2 + · · · + xn
MA = ,
n
√
MG = n x1 x2 · · · xn ,
n
MH = .
1 1 1
+ +···+
x1 x2 xn
Teorema 4.1
Sean x1 , x2 , . . . , xn números reales positivos entonces su Media Aritmética es mayor
o igual que su Media Geométrica (MA ≥ MG).
La igualdad ocurre si y sólo si x1 = x2 = · · · = xn .
55
56 4. Desigualdad de la Media Aritmética - Media Geométrica
x1 + x2 + · · · + xn−1 + g √
≥ n x1 x2 · · · xn−1 · g
n
x1 + x2 + · · · + xn−1 + g p
⇔ ≥ n g n−1 · g
n
√
⇔ x1 + x2 + · · · + xn−1 + g ≥ n · n g n = ng
⇔ x1 + x2 + · · · + xn−1 ≥ g(n − 1)
x1 + x2 + · · · + xn−1
⇔ ≥ g.
n−1
De donde
x1 + x2 + · · · + xn−1 √
≥ n−1
x1 x2 · · · xn−1 .
n−1
√ √ √ q√
x3 x4 + · · · + x2n−1 x2n
x1 x2 + √ √
≥ n x1 x2 · x3 x4 · · · x2n−1 x2n
n q
√ √ √ √
⇔ x1 x2 + x3 x4 + · · · + x2n−1 x2n ≥ n n x1 x2 x3 x4 · · · x2n−1 x2n
√ √ √ √
⇔ 2 x1 x2 + x3 x4 + · · · + x2n−1 x2n ≥ 2n 2n x1 x2 x3 x4 · · · x2n−1 x2n . (4.2)
√
x1 + x2 + · · · + x2n ≥ 2n 2n x1 x2 · · · x2n
x1 + x2 + · · · + x2n √
⇔ ≥ 2n x1 x2 · · · x2n ,
2n
1 1 1
Prueba. Aplicando el teorema 4.1 a los números , ,..., reales positivos,
x1 x2 xn
tenemos:
1 1 1 r
+ +···+
x1 x2 xn 1 1 1
≥ n · ··· ,
n x1 x2 xn
58 4. Desigualdad de la Media Aritmética - Media Geométrica
1 1 1 n
⇔ + +···+ ≥ √
x1 x2 xn n
x1 x2 · · · xn
√ n
⇔ n x1 x2 · · · xn ≥ ,
1 1 1
+ +···+
x1 x2 xn
MA ≥ MG ∧ MG ≥ MH ⇒ MA ≥ MH.
4.1. Aplicaciones
1. Sean a, b, c números reales positivos. Pruebe que
a b c
+ + ≥ 3.
b c a
4.1. Aplicaciones 59
a b c r
+ +
b c a ≥ 3 a·b·c
3 b c a
a b c √
+ + ≥ 3· 31
b c a
a b c
+ + ≥ 3.
b c a
a4 b4 c4
+ + ≥ 3abc.
b c a
a4 b4 c4
Prueba. Utilizando MA - MG, a los números , , .
b c a
En efecto
a4 b4 c4 r
+ + 4 4
b c a ≥ 3 a4 b · b · c
3 c a
4 4 4
a b c
+ + ≥ 3abc.
b c a
3. Sabiendo que a, b, c, d son números reales positivos con abcd = 1, pruebe que
a2 + b2 + c2 + d2 + ab + ac + ad + bc + bd + cd ≥ 10.
60 4. Desigualdad de la Media Aritmética - Media Geométrica
a2 + b2 + c2 + d2 + ab + ac + ad + bc + bd + cd p
≥ 10 (abcd)4
10
2 2 2 2
√
a + b + c + d + ab + ac + ad + bc + bd + cd ≥ 10 · 10 1
∴ a2 + b2 + c2 + d2 + ab + ac + ad + bc + bd + cd ≥ 10.
4. (Australia 2000). Sea a un número real no nulo, b un número real, pruebe que
1 b √
a2 + b2 + + ≥ 3.
a2 a
1 b
a2 + b2 + 2 +
a a
2 1 b 2 3
= b + 2+ + a + 2
4a a 4a
2
1 3
= b+ + a2 + 2
2a 4a
y además
r √
3 3 2 3 √
a2 + 2 ≥ 2 a2 · 2 = = 3
4a 4a 2
3 √
⇔ a2 + 2 ≥ 3
4a
2 2
1 2 3 1 √ √
⇔ b+ + a + 2 ≥ b+ + 3 ≥ 3.
2a 4a 2a
1 b √
∴ a2 + b2 + 2
+ ≥ 3.
a a
1
La igualdad ocurre si y sólo si b = − . 2
2a
4.1. Aplicaciones 61
5. Pruebe que
a2 + 2b ≥ 3(ab)2/3
a2 + b + b √
3
≥ a2 · b · b
3
√3
⇔ a2 + 2b ≥ 3 a2 b2
⇔ a2 + 2b ≥ 3(ab)2/3 .
4
f (x) = x2 + √ , x > 0.
x
4 1 1 1 1
f (x) = x2 + √ = x2 + √ + √ + √ + √
x x x x x
1 1 1 1
entonces aplicamos MA ≥ MG a los números x2 , √ , √ , √ , √ , veamos:
x x x x
1 1 1 1
x2 + √ + √ + √ + √ r
x x x x 1 1 1 1
≥ 5 x2 · √ · √ · √ · √
5 x x x x
4
⇔ x2 + √ ≥ 5(1)
x
entonces f (x) ≥ 5.
Por lo tanto, el mı́nimo de f es 5 y ocurre si y sólo si x = 1. 2
62 4. Desigualdad de la Media Aritmética - Media Geométrica
7. Pruebe que
r r r
2+1 3 3+1 n n+1
M =1+ + +···+ < n + 1.
2 3 n
r
k k+1
Prueba. El término general de la suma del primer miembro es
k
y vemos que
(k − 1) sumandos
r v k+1 z }| {
k+1 uk + 1 +1+1+···+1
u
k
= t
k · 1| · 1{z· · · 1} < k
k k k
(k − 1) factores
r k+1
k+1 +k−1 1
⇔ k
< k = 1 + 2.
k k k
En efecto
1 1 1
M < 1+ 1+ 2 + 1+ 2 +···+ 1+ 2
2 3 n
1 1 1
= n+ 2 + 2 +···+ 2
2 3 n
1 1 1
< n+ + +···+
1·2 2·3 (n − 1) · n
1 1 1 1 1 1
= n+ − + − +···+ −
1 2 2 3 n−1 n
1
= n + 1 − < n + 1. [10, pág 17]
n
2
8. Pruebe que
r √ r √
n
n
n n
n
n
1+ + 1− < 2,
n n
Prueba.
√
r √ v √
n
n √
u 1 + + (n − 1)
n
n
n u n
n n
n
n
1+ = t 1+
n · 1| · 1{z· · · 1} < =1+ 2 .
n n n n
(n − 1) factores
Similarmente
r √ √
n n n n
n
1− <1− 2 ,
n n
entonces
r √ r √ √ √
n n
n
n n
n n
n n
n
1+ + 1− < 1 + 2 + 1 − 2 = 2.
n n n n
r √ r √
n
n
n n
n
n
∴ 1+ + 1− < 2. [10, pág 17]
n n
2
p 1 3a + b + 2c + 3
⇔ a(b + 2c) ≤ √
3
; similarmente
3
9 3
p 1 3b + c + 2a + 3
3
b(c + 2a) ≤ √ ;
3
9 3
p 1 3c + a + 2b + 3
3
c(a + 2b) ≤ √ .
3
9 3
p3
p p 1 6(a + b + c) + 9
a(b + 2c) + 3 b(c + 2a) + 3 c(a + 2b) ≤ √ ,
| {z } 3
9 3
f (a,b,c)
1 √
3
⇒ f (a, b, c) ≤ √
3
· 9 = 3 3.
9
√
∴ máx f = 3 3 3. [10, pág 16] 2
x y z 2x y 2y z
Prueba. Observar que 3 + + = + + + +
y z x y z z x
2z x
+ .
x y
Utilizando MA ≥ MG, tenemos que
r
2x y x x y x x y
+ = + + ≥ 33 · · , entonces
y z y y z y y z
s
2x y x2
+ ≥ 33 , similarmente
y z yz
4.1. Aplicaciones 65
r
2
2y z 3 y
+ ≥ 3 ;
z x xz
s
2z x z2
+ ≥ 33
x y xy
s r s
x y z 3 x2 3 y2 3 z2
3 + + ≥ 3· +3· +3·
y z x yz xz xy
x y z
= 3 √3 xyz
+ √3 xyz
+ √
3 xyz
x y z x+y+z
⇔ + + ≥ √ ; similarmente
y z x 3 xyz
x z y x+y+z
+ + ≥ √ ,
z y x 3 xyz
x y z x z y 2(x + y + z)
+ + + + + ≥ √ ,
y z x z y x 3 xyz
x y z x y x z y z x y z 2(x + y + z)
1+ + + + · + · + · + · · ≥2+ √ ,
y z x y z y x z x y z x 3 xyz
| {z }
1
de donde
x y z 2(x + y + z)
1+ 1+ 1+ ≥2+ √ .
y z x 3 xyz
1 1 1 9
√ +√ +√ ≥ . (4.3)
a+b−c b+c−a c+a−b ab + bc + ca
Prueba. Haciendo
√ √ √
x= a + b − c; y = b + c − a; z = c + a − b,
x2 + z 2 x2 + y 2
entonces x2 + y 2 + z 2 = a + b + c = 3 y a = , b = ,
2 2
y2 + z2
c= , de donde se deduce que:
2
9 + x2 y 2 + y 2z 2 + z 2 x2
ab + ac + bc = .
4
1 1 1 36
+ + ≥
x y z 9 + x y + y 2z 2 + z 2 x2
2 2
p
⇔ (yz + xz + xy) (9 + (xy)2 + (yz)2 + (zx)2 ) ≥ 36xyz = 36 (xyz)2 .
Haciendo xy = m, xz = n, yz = p, tenemos
√
(m + n + p)(9 + m2 + n2 + p2 ) ≥ 36 mnp.
√
m + n + p ≥ 3 3 mnp (4.4)
9 sumandos
z }| {
m2 + n2 + p2 + 1 + 1 + · · · + 1 p
≥ 12 m2 n2 p2
12 p
m2 + n2 + p2 + 9 ≥ 12 12 (mnp)2
√
m2 + n2 + p2 + 9 ≥ 12 6 mnp. (4.5)
4.1. Aplicaciones 67
de donde obtenemos:
√
(m + n + p)(m2 + n2 + p2 + 9) ≥ 36 mnp.
n−1
n(x − y)(xy) 2 ≤ xn − y n .
n−1 xn − y n
n(xy) 2 ≤ ; x>y
x−y
n−1
n(xy) 2 ≤ xn−1 + xn−2 y + xn−3 y 2 + · · · + xy n−2 + y n−1.
Aplicando MA ≥ MG
2
68 4. Desigualdad de la Media Aritmética - Media Geométrica
CAPÍTULO 5
DESIGUALDAD DE BERNOULLI Y LA
MEDIA POTENCIAL
Teorema 5.1
Si x ≥ −1 y n entero positivo, entonces
(1 + x)n ≥ 1 + nx.
Si n = 1: 1 + x ≥ 1 + x, es verdadera.
(1 + x)k (1 + x) ≥ (1 + kx)(1 + x)
⇔ (1 + x)k+1 ≥ 1 + (k + 1)x + kx2 ≥ 1 + (k + 1)x
69
70 5. Desigualdad de Bernoulli y la Media Potencial
de donde
(1 + x)k+1 ≥ 1 + (k + 1)x.
∴ (1 + x)n ≥ 1 + nx.
2
Teorema 5.2
Si x ≥ −1 y 0 < α < 1, entonces
(1 + x)α ≤ 1 + αx.
m p p
(1 + x)α = (1 + x) n = n
(1 + x)m = n
(1 + x)m · 1n−m
s
= (1 + x) · (1 + x) · · · (1 + x) · 1| · 1{z· · · 1}
n | {z }
m n−m
m n−m
z }| { z }| {
(1 + x) + (1 + x) + · · · + (1 + x) + 1 + 1 + · · · + 1
≤
m + (n − m)
m(1 + x) + n − m n + mx m
= = = 1 + x = 1 + αx,
n n n
de donde se tiene
(1 + x)qk ≤ 1 + qk x; x ≥ −1, k = 1, 2, 3, . . .
71
luego
Teorema 5.3
Si x ≥ −1 y (α < 0 ∨ α > 1), se tiene
(1 + x)α ≥ 1 + αx.
1 1
(1 + αx) α ≤ 1 + · αx = 1 + x,
α
de donde
Sea α < 0, tomemos un entero n positivo tal que − αn < 1, luego por el teorema
5.2
α
−α
(1 + x) n ≤ 1+ − x
n
α 1 α
⇔ (1 + x) n ≥ α ≥ 1 + n x.
1− x
n
Teorema 5.4
Si x1 , x2 , . . . , xn números reales positivos y α < 0 < β, entonces
Mα ≤ MG ≤ Mβ .
α α
1
α α
√ x 1 + x 2
1+ · · · + xn
( xα1 · xα2 · · · xαn ) ≥
n α
n
α 1
√ x1 + xα2 + · · · + xαn α
⇔ n
x1 · x2 · · · xn ≥ = Mα ,
n
de donde Mα ≤ MG.
Ası́ mismo
q
xβ1 + xβ2 + · · · + xβn
xβ1 xβ2 · · · xβn
n
· ≤ ; β > 0,
n
1
elevando a la potencia β
q β1 ! β1
n β β β xβ1 + xβ2
+···+ xβn
x1 · x2 · · · xn ≤
n
! β1
√ xβ1 + xβ2 + · · · + xβn
⇔ n
x1 · x2 · · · xn ≤ = Mβ ,
n
de donde MG ≤ Mβ .
∴ Mα ≤ MG ≤ Mβ .
2
Teorema 5.5
Si x1 , x2 , . . . , xn son números reales positivos y α < β, se tiene
Mα ≤ Mβ .
entonces
β β β β1
x1 x2 xn
+ +···+
Mβ Mβ q q q
= = .
Mα q n
Tomando
α α α
x1 x2 xn
d1 = ; d2 = ; · · · ; dn = ,
q q q
obtenemos
β β β
β1
Mβ d1 + d2 + · · · + dn
α α α
= .
q n
Pero
α α α α1
x1 x2 xn
α1
q + +···+
d1 + d2 + · · · + dn q q
=
n n
α1
1 xα1 + xα2 + · · · + xαn 1
= · = · q = 1,
q n q
d1 + d2 + · · · + dn
resulta = 1, es decir
n
d1 + d2 + · · · + dn = n.
Haciendo
d1 = 1 + k1 ; d2 = 1 + k2 ; . . . ; dn = 1 + kn ,
entonces k1 + k2 + · · · + kn = 0.
β
Como > 1, pues α < β, se tiene
α
5.1. Media Potencial 75
β β β
α1α = (1 + k1 ) α ≥ 1+ k1
α
β β β
α2α = (1 + k2 ) α ≥ 1 + k2
α
.. ..
. .
β β β
αnα = (1 + kn ) α ≥ 1 + kn .
α
Sumando miembro a miembro, tenemos
β β β β
α1α + α2α + · · · + αnα ≥ n + (k1 + k2 + · · · + kn )
α | {z }
0
β β β
⇔ α1 + α2 + · · · + αn
α α α
≥ n
β β β
α α + α2α + · · · + αnα
⇔ 1 ≥ 1
n
β β β
β1
α + α2 + · · · + αn
α α α
⇔ 1 ≥ 1,
n
de donde
Mβ
≥ 1 ⇔ Mβ ≥ q ⇔ Mβ ≥ Mα .
q
La igualdad Mβ = Mα se cumple si y sólo si
k1 = k2 = · · · = kn = 0, luego
d1 = d2 = · · · = dn = 1, y por consiguiente
x1 = x2 = · · · = xn .
β
Falta demostrar el caso α < β < 0, pero de α < β < 0 se tiene 0 < < 1, y se
α
repite el caso anterior.
∴ Mα ≤ Mβ .
2
76 5. Desigualdad de Bernoulli y la Media Potencial
5.2. Aplicaciones
1. Demuestre que si x3 + y 3 + z 3 = 2187, siendo x, y, z números reales positivos,
entonces
x + y + z ≤ 27.
⇔ (x + y + z)3 ≤ 32 · 37 = 39
⇔ x + y + z ≤ 33 .
Solución. Como x, y, z > 0 y tenemos que buscar una relación entre x+y+z
y x3 + y 3 + z 3 , entonces aplicamos la Media Potencial, veamos:
1/3
x+y+z x3 + y 3 + z 3
≤
3 3
(x + y + z)3 x3 + y 3 + z 3
⇔ ≤
27 3
√3 3
( 3)
⇔ ≤ x3 + y 3 + z 3
9
1
⇔ ≤ x3 + y 3 + z 3
3
√
3
1 3
Por lo tanto el mı́nimo de f es y ocurre si y sólo si x = y = z = . 2
3 3
5.2. Aplicaciones 77
x5 + y 5 + z 5 ≥ 5(x + y + z) − 12.
Haciendo 1 + a = x ⇒ a = x − 1, entonces
x5 ≥ 1 + 5(x − 1)
⇔ x5 ≥ 5x − 4; x ≥ 0; similarmente
y 5 ≥ 5y − 4
z 5 ≥ 5z − 4
x5 + y 5 + z 5 ≥ 5x + 5y + 5z − 12.
∴ x5 + y 5 + z 5 ≥ 5(x + y + z) − 12.
ck = y1 + y2 + · · · + yk ; (k = 1, 2, . . . n),
entonces
my1 ≤ x1 y1 + x2 y2 + · · · + xn yn ≤ My1 .
79
80 6. Desigualdad de Reordenamientos
de donde
n
X
xi yi ≥ my1 .
i=1
Similarmente
n
X n
X
xi yi = (yi − yi+1 )si ; yn+1 = 0
i=1 i=1
n
X n
X
≤ (yi − yi+1 )M = M (yi − yi+1 ) = My1 ,
i=1 i=1
de donde
n
X
xi yi ≤ My1 .
i=1
n
X
∴ my1 ≤ xi yi ≤ My1 .
i=1
2
Aplicación 6.1
Sean a1 , a2 , · · · , an y b1 ≥ b2 ≥ · · · ≥ bn ≥ 0 números reales positivos tales que
a1 · a2 · · · ak ≥ b1 · b2 · · · bk , ∀ k ∈ {1, 2, . . . , n}.
Pruebe que
a1 + a2 + · · · + an ≥ b1 + b2 + · · · + bn .
81
n
X n
X n
X
ai
ai − bi = bi −1
i=1 i=1 i=1
b i
a1 a1 a2
= (b1 − b2 ) − 1 + (b2 − b3 ) + −2 +···
b1 b1 b2
n−1
! n
!
X ai X ai
+(bn−1 − bn ) − n + 1 + bn −n .
b
i=1 i
b
i=1 i
k
X ai
Analizando cada suma ; k = 1, 2, . . . , n, mediante la propiedad MA ≥ MG
i=1
bi
r
a1 a2 ak a1 a2 ak
+ +···+ ≥k k
· ··· ≥ k,
b1 b2 bk b1 b2 bk
luego
k
!
X ai
−k ≥ 0.
i=1
bi
n
X n
X
∴ ai − bi ≥ 0.
i=1 i=1
2
Teorema 6.3 (Desigualdad de Reordenamientos)
Sean (a1 , a2 , . . . , an ) y (b1 , b2 , . . . , bn ) dos sucesiones crecientes de números reales y
sea (bi1 , bi2 , . . . , bin ) una permutación de (b1 , b2 , . . . , bn ), entonces
a1 ≤ a2 ≤ · · · ≤ an y b1 ≤ b2 ≤ · · · ≤ bn .
82 6. Desigualdad de Reordenamientos
n
X n
X n
X
ak bk − ak bik = ak (bk − bik )
k=1 i=1 k=1
ya que b1 ≤ b2 ≤ · · · ≤ bn .
La segunda parte del teorema se prueba similarmente. 2
Aplicación 6.2
Pruebe que
a b c 3
+ + ≥
b+c c+a a+b 2
para todo a, b, c números reales positivos.
Luego
a b c b c a
+ + ≥ + +
b+c c+a a+b b+c c+a a+b
a b c c a b
+ + ≥ + + ,
b+c c+a a+b b+c c+a a+b
sumando miembro a miembro, tenemos:
a b c b+c c+a a+b
2 + + ≥ + + = 3,
b+c c+a a+b b+c c+a a+b
de donde
a b c 3
+ + ≥ .
b+c c+a a+b 2
2
Colorario 6.1 Para toda permutación (a′1 , a′2 , . . . , a′n ) de (a1 , a2 , . . . , an ), se tiene
Colorario 6.2 Para toda permutación (a′1 , a′2 , . . . , a′n ) de (a1 , a2 , . . . , an ), se tiene
a1 b1 + a2 b2 + · · · + an bn a1 + a2 + · · · + an b1 + b2 + · · · + bn
≥ .
n n n
a1 b1 + a2 b2 + · · · + an bn a1 + a2 + · · · + an b1 + b2 + · · · + bn
≤ .
n n n
84 6. Desigualdad de Reordenamientos
Prueba. Veamos el caso cuando las dos sucesiones son crecientes. En efecto,
por la desigualdad de reordenamientos, tenemos
a1 b1 + a2 b2 + · · · + an bn = a1 b1 + a2 b2 + · · · + an bn ,
a1 b1 + a2 b2 + · · · + an bn ≥ a1 b2 + a2 b3 + · · · + an b1 ,
a1 b1 + a2 b2 + · · · + an bn ≥ a1 b3 + a2 b4 + · · · + an b2 ,
.. .. ..
. . .
a1 b1 + a2 b2 + · · · + an bn ≥ a1 bn + a2 b1 + · · · + an bn−1 .
a3 b3 c3 (a + b + c)3
+ + ≥
x y z 3(x + y + z)
a2 b2 c2
como a ≥ b ≥ c y z ≥ y ≥ x, entonces ≥ ≥ .
x y z
Aplicando el teorema de Chebyshev
2
a2 b2 c2 a b2 c2
·a+ ·b+ ·c
x y z x + y + z a+b+c
≥
3 3 3
a3 b3 c3 a2 b2 c2 a+b+c
⇔ + + ≥ + + .
x y z x y z 3
85
a2 b2 c2 (a + b + c)2
+ + ≥ .
x y z x+y+z
a+b+c
Multiplicando por , tenemos
3
2
a b2 c2 a+b+c (a + b + c)3
+ + ≥ .
x y z 3 3(x + y + z)
2
Aplicación 6.4
Pruebe que
xα yα zα 3
+ + ≥
y+z z+x x+y 2
para todo α, x, y, z números reales positivos tales que xyz = 1 y α ≥ 1.
Utilizando Chebyshev
xα yα zα 1 α−1 x y z
+ + ≥ x + y α−1 + z α−1 + + .
y+z z+x x+y 3 y+z z+x x+y
86 6. Desigualdad de Reordenamientos
xα yα zα 3
∴ + + ≥ .
y+z z+x x+y 2
2
Aplicación 6.5
Pruebe que
a3 b3 c3 d3 1
+ + + ≥
b+c+d a+c+d a+b+d a+b+c 3
para todo a, b, c, d reales positivos con ab + bc + cd + da = 1.
m = b + c + d,
n = a + c + d,
p = a + b + d,
q = a+b+c
1 1 1 1
⇒ m≤n≤p≤q ∧ ≥ ≥ ≥ ,
m n p q
a3 b3 c3 d3
+ + +
m n p q
1 3 3 3 3 1 1 1 1
≥ (a + b + c + d ) + + +
4 m n p q
1 2 2 2 2 1 1 1 1
≥ (a + b + c + d ) (a + b + c + d) + + +
16 m n p q
87
a3 b3 c3 d3
+ + +
m n p q
1 2 m + n + p + q 1 1 1 1
≥ (a + b2 + c2 + d2 ) + + +
16 3 m n p q
1 2 2 2 2 1 1 1 1 1
= (a + b + c + d ) (m + n + p + q) + + +
16 3 m n p q
1 2 1
≥ (a + b2 + c2 + d2 ) (16)
16 3
1 2
= (a + b2 + c2 + d2 ) .
3
a3 b3 c3 d3 1
∴ + + + ≥ .
m n p q 3
2
88 6. Desigualdad de Reordenamientos
CAPÍTULO 7
DESIGUALDAD CON FUNCIONES
CONVEXAS
Teorema 7.1
Si f es una función real definida sobre [a, b] ⊂ R y f ′′ (x) > 0 para todo x ∈ ha, bi,
entonces f es una función convexa sobre [a, b].
89
90 7. Desigualdad con Funciones Convexas
Prueba. Debemos probar que para todo x ∈ [a, b] y para todo t ∈ [0, 1], se
cumple
Como f ′′ (x) > 0, ∀ x ∈ [a, b], entonces f ′ es creciente en [a, b], de donde tenemos
que
g ′(x) ≥ 0 si x ≥ y; y
g ′(x) ≤ 0 si x ≤ y,
de donde
Reemplazando
∴ f (tx + (1 − t)y) ≤ tf (x) + (1 − t)f (y), ∀ x ∈ [a; b], ∀ t ∈ [0; 1]. [9]
2
7.1. Función convexa 91
Ejemplo 7.1
La función f (x) = ǫx , con x número real, es convexa.
−4 −3 −2 −1 1 2 3 4
−1
1
Prueba. Es suficiente tomar t = en la definición. 2
2
Definición 7.2 : Una función f real definida sobre [a, b] ⊂ R es llamada función
cóncava sobre [a, b] si y sólo si para cada x, y ∈ [a, b] y para todo 0 ≤ t ≤ 1 se tiene
h i
t1 t2 tn
t1 x1 + t2 x2 + · · · + tn−1 xn−1 + tn xn = (1 − tn ) x
1−tn 1
+ x
1−tn 2
+···+ x
1−tn−1 n−1
+tn xn ,
entonces
t1 tn−1
f (t1 x1 + t2 x2 + · · · + tn xn ) = f (1 − tn ) x1 + · · · + xn−1 + tn xn
1 − tn 1 − tn
t1 tn−1
≤ (1 − tn )f x1 + · · · + xn−1
1 − tn 1 − tn
+tn f (xn ), f convexa
t1 tn−1
≤ (1 − tn ) f (x1 ) + · · · + f (xn−1 )
1 − tn 1 − tn
+tn f (xn ),
= t1 f (x1 ) + t2 f (x2 ) + · · · + tn f (xn ),
de donde se tiene
2
1
En particular, si t1 = t2 = · · · = tn = , se tiene
n
x1 + x2 + · · · + xn 1
f ≤ (f (x1 ) + f (x2 ) + · · · + f (xn )) .
n n
Similarmente si f es cóncava entonces
Teorema 7.4
Si f es una función real definida sobre [a, b] ⊂ R, y f ′′ (x) < 0 para todo x ∈ ha, bi,
entonces f es una función cóncava sobre [a, b].
7.1. Función convexa 93
entonces
Prueba. Por la forma de cada fracción del primer miembro, definimos la función
1
f (x) = ; x ∈ R+ .
1 + ǫx
Veamos que es convexa.
Derivando, tenemos
−ǫx
f ′ (x) = y
(1 + ǫx )2
′′ ǫx (ǫx − 1)
f (x) = > 0, ∀ x > 0.
(1 + ǫx )3
Por lo tanto f es convexa. Luego, podemos aplicar la Desigualdad de Jensen
x1 + x2 + · · · + xn 1
f ≤ (f (x1 ) + f (x2 ) + · · · + f (xn ))
n n
1 1 1 1 1
x1 +x2 +···+xn ≤ + +···+
1+ǫ n n 1 + ǫx1 1 + ǫx2 1 + ǫxn
1
Prueba. Definimos la función f (x) = √ , x > 0. La función es convexa, y sin
x
pérdida de generalidad asumimos que a + b + c = 1, luego
Una de las operaciones principales del análisis es el paso al lı́mite. Esta operación
descansa sobre el hecho de que en la recta numérica está definida la distancia entre
dos puntos.
Es impresionante ver que muchos resultados principales del análisis no tienen
nada que ver con la naturaleza algebraica del conjunto de los números reales, es
decir, sólo se apoya en las propiedades de distancia y con ello llegamos al concepto
de Espacio Métrico, que es uno de los conceptos más importantes de la matemática
moderna.
d : X × X −→ R
(x, y) 7−→ d(x, y)
(1) d(x, y) = 0 ⇔ x = y.
97
98 8. Espacio Métrico
d(x, y) = |x − y|
Prueba.
(1)
d(x, y) = 0 ⇔ |x − y| = 0
⇔ x−y =0
⇔ x = y.
3. El conjunto
Rn = {x = (x1 , x2 , . . . , xn )/ x1 , x2 , . . . , xn ∈ R}
99
con la distancia
v
u n
uX
d(x, y) = t (yk − xk )2
k=1
n
!2 n n
X X X
ak bk ≤ a2k · b2k . (8.3)
k=1 k=1 k=1
100 8. Espacio Métrico
En efecto:
n
X n
X n
X n
X
2
(ak + bk ) = a2k +2 ak bk + b2k
k=1 k=1 k=1 k=1
v
n u n n n
X uX X X
≤ 2
ak + 2 t 2
ak · 2
bk + b2k
k=1 k=1 k=1 k=1
v v 2
u n u n
uX uX
= t a2k + t b2k ,
k=1 k=1
de donde
v v v
u n u n u n
uX uX uX
t (ak + bk )2 ≤ t a2k + t b2k .
k=1 k=1 k=1
n
X
d1 (x, y) = |yk − xk |.
k=1
n
!1/p
X
dp (x, y) = |yk − xk |p ,
k=1
donde p es un número fijo arbitrario mayor que 1 (p > 1), representa el espacio
métrico (Rn , dp ). Para comprobar que es espacio métrico, veamos que cumple
los tres axiomas.
(1) y (2) son obvias, veamos el axioma triangular (3), es decir, se debe cumplir
n
!1/p n
!1/p n
!1/p
X X X
|zk − xk |p ≤ |zk − yk |p + |yk − xk |p .
k=1 k=1 k=1
Haciendo
zk − yk = ak y yk − xk = bk ⇒ zk − xk = ak + bk ,
tenemos
n
!1/p n
!1/p n
!1/p
X X X
|ak + bk |p ≤ |ak |p + |bk |p .
k=1 k=1 k=1
n
X n
X
Prueba. Supongamos que xai = yib = 1, usando el hecho que
i=1 i=1
1 1
xi yi ≤ xai + yib (ejercicio demostrado anteriormente),
a b
entonces
n
X n n
1X a 1X b 1 1
xi yi ≤ x + y = + = 1. (8.4)
i=1
a i=1 i b i=1 i a b
n
X n
X
Ahora supongamos que xai =M y yib = N.
i=1 i=1
xi yi
Sea x′i = 1/a , yi′ = 1/b , entonces
M N
Xn
n
xai
X a i=1 M
x′ i = = = 1,
i=1
M M
n
X
n
yib
X b i=1 N
y′i = = = 1.
i=1
N N
Luego
n
X n
X Xn
′ ′ xi yi 1
x iy i = = 1/a xi yi. (8.5)
i=1 i=1
M 1/a · N 1/b M · N 1/b i=1
8.2. Desigualdad de Minkonski 103
n
X
De (8.4) se tiene que x′i yi′ ≤ 1, entonces de (8.5) tenemos
i=1
X n
1
xi yi ≤ 1
M 1/a · N 1/b i=1
n
X
xi yi ≤ M 1/a · N 1/b .
i=1
Reemplazando
n n
!1/a n
!1/b
X X X
xi yi ≤ xai · yib .
i=1 i=1 i=1
n
!1/p n
!1/p n
!1/p
X X X
(ak + bk )p ≤ (ak )p + (bk )p .
k=1 k=1 k=1
Prueba. Como
entonces
n
X n
X n
X
(ak + bk )p = ak (ak + bk )p−1 + bk (ak + bk )p−1 , (8.6)
k=1 k=1 k=1
n n
!1/p n
!1/q
X X X
ak (ak + bk )p−1 ≤ (ak )p · (ak + bk )q(p−1) ,
k=1 k=1 k=1
n n
!1/p n
!1/q
X X X
bk (ak + bk )p−1 ≤ (bk )p · (ak + bk )q(p−1) ,
k=1 k=1 k=1
n n
!1/p n
!1/q
X X X
(ak + bk )p ≤ (ak )p · (ak + bk )p
k=1 k=1 k=1
n
!1/p n
!1/q
X X
+ (bk )p · (ak + bk )p
k=1 k=1
n n
!1/q n !1/p n
!1/p
X X X X
(ak + bk )p ≤ (ak + bk )p · (ak )p + (bk )p
k=1 k=1 k=1 k=1
n
!1−1/q n
!1/p n
!1/p
X X X
(ak + bk )p ≤ (ak )p + (bk )p
k=1 k=1 k=1
n
!1/p n
!1/p n
!1/p
X X X
(ak + bk )p ≤ (ak )p + (bk )p .
k=1 k=1 k=1
2
Teorema 8.1
Dados los puntos A, B, C del plano, se tiene que
AB ≤ AC + CB,
a b c 2a 2b 2c
+ + = + +
b+c c+a a+b 2b + 2c 2c + 2a 2a + 2b
2a 2b 2c
= + +
(b + c) +(b + c) (c + a) +(c + a) (a + b) +(a + b)
| {z } | {z } | {z }
2a 2b 2c
< + +
a+b+c a+b+c a+b+c
2(a + b + c)
= = 2,
a+b+c
de donde
a b c
+ + < 2.
b+c c+a a+b
2
Aplicación 8.2
Pruebe que
√ p p √
2|x + y + z| ≤ x2 + y 2 + y 2 + z 2 + x2 + z 2 ,
Pero
de donde
√ p p √
2|x + y + z| ≤ x2 + y 2 + y 2 + z 2 + x2 + z 2 .
2
106 8. Espacio Métrico
Aplicación 8.3
Pruebe que
x4/3 y 4/3
M = +
x4/3 + (x2 + y 2)1/3 (x + z)2/3 y 4/3 + (y 2 + z 2 )1/3 (y + z)2/3
z 4/3
+ ≤1
z 4/3 + (z 2 + x2 )1/3 (x + y)2/3
1/3 1/3
(x2 + y 2 ) (x + z)2/3 = ((a2 )3 + (b2 )3 ) ((c2 )3/2 + (a2 )3/2 )2/3
≥ a2 c2 + b2 a2 = (xy)2/3 + (xz)2/3
1/3
⇒ x4/3 + (x2 + y 2 ) (x + z)2/3 ≥ x4/3 + (xy)2/3 + (xz)2/3
x4/3 x4/3
⇒ ≤
x4/3 + (x2 + y 2 )1/3 (x + z)2/3 x4/3 + (xy)2/3 + (xz)2/3
x2/3
= 2/3
x + y 2/3 + z 2/3
similarmente
y 4/3 y 2/3
≤ ,
y 4/3 + (y 2 + z 2 )1/3 (y + z)2/3 x2/3 + y 2/3 + z 2/3
z 4/3 z 2/3
≤ ,
z 4/3 + (z 2 + x2 )1/3 (z + x)2/3 x2/3 + y 2/3 + z 2/3
M ≤ 1.
2
8.2. Desigualdad de Minkonski 107
Aplicación 8.4
Sean a, b, c, d números reales. Determine el mı́nimo de
p p
F = (a + 1)2 + 2(b − 2)2 + (c + 3)2 + (b + 1)2 + 2(c − 2)2 + (d + 3)2
p p
+ (c + 1)2 + 2(d − 2)2 + (a + 3)2 + (d + 1)2 + 2(a − 2)2 + (b + 3)2 .
p p
S1 = (a + 1)2 + 2(b − 2)2 + (c + 3)2 + (b + 1)2 + 2(c − 2)2 + (d + 3)2
p
≥ (a + 1 + b + 1)2 + 2(b − 2 + c − 2)2 + (c + 3 + d + 3)2
p
= (a + b + 2)2 + 2(b + c − 4)2 + (c + d + 6)2
p p
S2 = (c + 1)2 + 2(d − 2)2 + (a + 3)2 + (d + 1)2 + 2(a − 2)2 + (b + 3)2
p
≥ (c + d + 2)2 + 2(a + d − 4)2 + (a + b + 6)2
p
S1 + S2 ≥ (a + b + 2)2 + 2(b + c − 4)2 + (c + d + 6)2
p
+ (c + d + 2)2 + 2(a + d − 4)2 + (a + b + 6)2
(aplicando nuevamente Minkowski)
p
F ≥ (a + b + c + d + 4)2 + 2(a + b + c + d − 8)2 + (a + b + c + d + 12)2
p
= (s + 4)2 + 2(s − 8)2 + (s + 12)2
A ≥ B,
Si Sa , Sb , Sc ≥ 0, entonces
109
110 9. Método de la Suma de Cuadrados
Teorema 9.1
Si a, b, c, Sa , Sb , Sc satisfacen las condiciones
1. Sa + Sb ≥ 0, Sb + Sc ≥ 0, Sc + Sa ≥ 0, y
2. (a ≤ b ≤ c ∨ a ≥ b ≥ c) y Sb ≥ 0,
entonces
(a = b = c) ∨ (a = b ∧ Sa = Sb = 0) ∨ (b = c ∧ Sb = Sc = 0)
∨ (Sa = Sb = Sc = 0).
2
Teorema 9.2
Si a, b, c, Sa , Sb , Sc son números reales que satisfacen las condiciones:
(a ≤ b ≤ c ∨ a ≥ b ≥ c), Sa ≥ 0, Sc ≥ 0, Sa + 2Sb ≥ 0, Sc + 2Sb ≥ 0, entonces
Teorema 9.4
Si a, b, c, Sa , Sb , Sc son números reales que satisfacen las condiciones
1. Sa + Sb ≥ 0 ∨ S a + Sc ≥ 0 ∨ Sb + Sc ≥ 0, y
2. Sa Sb + Sb Sc + Sc Sa ≥ 0,
entonces
2
Aplicación 9.1
Pruebe que
a b c b c a 1 1 1
3 mı́n + + , + + ≥ (a + b + c) + + ,
b c a a b c a b c
√ √ √
siendo a, b, clos lados de un triángulo.
(pues (a − b) + (b − c) + (c − a) = 0)
⇔ (a − b)2 (c| + {z
a − }b) + (b − c)2 (a b − }c) + (c − a)2 (b| + {z
| +{z c − a}) ≥ 0
Sc Sa Sb
2 2 2
⇔ Sa (b − c) + Sb (c − a) + Sc (a − b) ≥ 0 (9.6)
y como
Sa + Sb = 2b > 0,
Sa + Sc = 2a > 0,
Sb + Sc = 2c > 0
entonces para que se cumpla (9.6) por el teorema 9.4 basta probar que
Sa Sb + Sa Sc + Sb Sc ≥ 0.
114 9. Método de la Suma de Cuadrados
Aplicación 9.2
Sean a, b, c reales positivos tales que ab + bc + ca = 1. Pruebe que
1 + a2 b2 1 + b2 c2 1 + c2 a2 5
2
+ 2
+ 2
≥ .
(a + b) (b + c) (c + a) 2
Prueba. Veamos
1 + a2 b2 12 + a2 b2 (ab + bc + ca)2 + a2 b2
= =
(a + b)2 (a + b)2 (a + b)2
2a2 b2 + 2ab(bc + ca) + (bc + ca)2
=
(a + b)2
2ab(ab + ac + bc) + (bc + ca)2 2ab(ab + ac + bc)
= 2
= + c2 ,
(a + b) (a + b)2
115
entonces
1 + a2 b2 2ab(ab + ac + bc)
2
= + c2 ; análogamente
(a + b) (a + b)2
1 + b2 c2 2bc(ab + ac + bc)
2
= + a2 ;
(b + c) (b + c)2
1 + c2 a2 2ca(ab + ac + bc)
2
= + b2 ;
(c + a) (c + a)2
1 + a2 b2 1 + b2 c2 1 + c2 a2 2ab 2bc 2ac
+ + = (ab + ac + bc) + +
(a + b)2 (b + c)2 (c + a)2 (a + b)2 (b + c)2 (a + c)2
+a2 + b2 + c2 ,
2ab 2bc 2ac 5
(ab + ac + bc) 2
+ 2
+ 2
+ a2 + b2 + c2 ≥
(a + b) (b + c) (a + c) 2
4ab 4bc 4ac
⇔ (ab + ac + bc) 2
+ 2
+ + 2(a2 + b2 + c2 )
(a + b) (b + c) (a + c)2
≥ 5 = 5(ab + bc + ac)
4ab 4bc 4ac
⇔ (ab + ac + bc) + + −3
(a + b)2 (b + c)2 (a + c)2
+2(a2 + b2 + c2 − ab − ac − bc) ≥ 0
4ab 4bc 4ac
⇔ (ab + ac + bc) −1 + −1 + −1
(a + b)2 (b + c)2 (a + c)2
+2(a2 + b2 + c2 − ab − ac − bc) ≥ 0
116 9. Método de la Suma de Cuadrados
(a − b)2 (b − c)2 (a − c)2
⇔ −(ab + ac + bc) + +
(a + b)2 (b + c)2 (a + c)2
+(a − b)2 + (b − c)2 + (c − a)2 ≥ 0
(ab + ac + bc) 2 (ab + ac + bc)
⇔ 1− (a − b) + 1 − (b − c)2
(a + b)2 (b + c)2
(ab + ac + bc)
+ 1− (c − a)2 ≥ 0.
(a + c)2
Haciendo
ab + ac + bc ab + ac + bc ab + ac + bc
Sa = 1 − ; S b = 1 − ; S c = 1 − .
(b + c)2 (a + c)2 (a + b)2
ab + ac + bc a2 + (a + c)(c − b)
Sb = 1 − = ≥ 0 ⇒ Sa ≥ Sb ≥ 0 y
(c + a)2 (a + c)2
2 2 2 ab + bc + ca 2 ab + bc + ca
b Sc + c Sb = b 1 − +c 1−
(a + b)2 (c + a)2
2 2
2 a + (a + b)(b − c) 2 a + (a + c)(c − b)
= b +c
(a + b)2 (c + a)2
2 2
2 2
b c c b
= a2 + + (c − b) −
(a + b)2 (c + a)2 (a + c) (a + b)
2 b2 c2 2 ab + bc + ca
= a + + (c − b) > 0,
(a + b)2 (c + a)2 (a + c)(a + b)
n
X
H(x) = (x + a1 )(x + a2 ) · · · (x + an ) = ck xn−k
k=0
c0 = 1
n
X
c1 = a1 + a2 + · · · + an = ai
i=1
X
c2 = a1 a2 + a1 a3 + · · · + an−1 an = ai aj
1≤i<j≤n
..
.
cn = a1 a2 · · · an .
117
118 10. Desigualdades Simétricas y Cı́clicas
Ejemplo 10.1
Si H(x) = (x + a)(x + b)(x + c) = x3 + (a + b + c)x2 + (ab + ac + bc)x + abc, entonces
c0 = 1
c1 = a + b + c
c2 = ab + ac + bc
c3 = abc.
1 k!(n − k)!
Pk =
n ck = ck ; k = 0, 1, 2, . . . , n.
k
n!
Ejemplo 10.2
P 0 = c0 = 1
1 a1 + a2 + · · · + an
P1 = c1 =
n n
2 2
P2 = c2 = (a1 a2 + a1 a3 + · · · + an−1 an ).
n(n − 1) n(n − 1)
n
X Xn
n−k n
H(x) = (x + a1 )(x + a2 ) · · · (x + an ) = ck x = Pk xn−k .
k=0 k=0
k
119
Derivando tenemos:
n−1
X
n
′
H (x) = (n − k) Pk xn−k−1 .
k=0
k
Definimos
n−1
X n−1
X
1 ′ n−k n n−k−1 n−1
Q(x) = H (x) = Pk x = Pk xn−k−1 .
n k=0
n k k=0
k
Si los valores reales ak , k = 1, 2, . . . , n son elementos del intervalo [α, β], entonces
el polinomio H(x) tiene n raı́ces reales en [α, β] y por el teorema de Rolle (consúltese
cualquier libro de análisis matemático) entonces H ′(x) tiene (n − 1) raı́ces reales en
[α, β] y denotemos como −y1 , −y2 , . . . , −yn−1 ; luego
1 ′
Q(x) = H (x) = (x + y1 )(x + y1 ) · · · (x + yn−1 ).
n
Igualando coeficientes de Q(x), obtenemos:
donde j = 2, 3, . . . , (n − 2).
Para j = 1 y n = 2, tenemos:
2
a1 + a2
P0 · P2 ≤ P12 ⇔ 1 · a1 a2 ≤
2
⇔ 4a1 a2 ≤ a21 + a22 + 2a1 a2
⇔ (a1 − a2 )2 ≥ 0
Para j = 1 y n = 3.
Si a1 a2 a3 = 0, se cumple trivialmente.
Si a1 a2 a3 6= 0, entonces dividiendo por (a1 a2 a3 )2 , tenemos
2
1 1 1 1 1 1 1 1
+ + ≤ + +
3 a1 a2 a2 a3 a3 a1 9 a1 a2 a3
2
1 1 1 1 1 1
⇔ 3 + + ≤ + +
a1 a2 a2 a3 a3 a1 a1 a2 a3
2 2 2
1 1 1 1 1 1
⇔ + + ≥ + + .
a1 a2 a3 a1 a2 a1 a3 a2 a3
n
!2
1X
≤ a1 a2 · · · abj · · · an
n j=1
k−1
Y 2(k−1)
Pkk−1 · k
Pk+1 Pj2j = P0 · P12 · P24 · P36 · · · Pk−1 · Pkk−1 · Pk+1
k
j=0
Pkk−1 · k
Pk+1 ≤ Pk2k
k
⇔ Pk+1 ≤ Pkk+1
1/k+1 1/k
⇔ Pk+1 ≤ Pk .
La igualdad ocurre si y sólo si se cumple a1 = a2 = · · · = an . 2
122 10. Desigualdades Simétricas y Cı́clicas
Aplicación 10.1
Sean a, b, c, d números reales positivos, pruebe que
27
(ab + ac + ad + bc + bd + cd)3 ≥ (abc + abd + acd + bcd)2 .
2
1/2 1/3
P2 ≥ P3 ⇔ P23 ≥ P32
!3 !2
1 1
⇔ 4 (ab + ac + ad + bc + bd + cd)
≥ 4 (abc + abd + acd + bcd)
2 3
1 1
⇔ 3
(ab + ac + ad + bc + bd + cd)3 ≥ 2 (abc + abd + acd + bcd)2
6 4
27
⇔ (ab + ac + ad + bc + bd + cd)3 ≥ (abc + abd + acd + bcd)2 .
2
Aplicación 10.2
Dados a, b, c, d números reales positivos, pruebe que
2
1 1 1 1 1 1 3 1 1 1 1
+ + + + + ≤ + + + .
ab ac ad bc bd cd 8 a b c d
2
1 2 1 1 1 1 1 3 2 1 1 1 1
(abcd) + + + + + ≤ (abcd) + + +
ab ac ad bc bd cd 8 a b c d
2
cd + bd + bc + ad + ac + ab bcd + acd + abd + abc
⇔ abcd ≤
6 4
⇔ P4 · P2 ≤ (P3 )2 ,
Definición 10.3 :
X
P (x, y, z) = P (x, y, z) + P (y, z, x) + P (z, x, y)
cı́c
X
P (x, y, z) = P (x, y, z) + P (x, z, y) + P (y, z, x)
sim
+P (y, x, z) + P (z, x, y) + P (z, y, x).
Ejemplo 10.3
X
x2 y = x2 y + y 2z + z 2 x
cı́c
X
x2 y = x2 y + x2 z + y 2 x + y 2 z + z 2 x + z 2 y
sim
X
x3 = x3 + y 3 + z 3
cı́c
X
x3 = 2 (x3 + y 3 + z 3 )
sim
X
xyz = xyz + yzx + zxy = 3xyz
cı́c
X
xyz = xyz + xzy + yxz + yzx + zxy + zyx = 6xyz.
sim
a1 ≥ a2 ≥ a3 , b1 ≥ b2 ≥ b3 , a1 ≥ b1 , a1 + a2 ≥ b1 + b2 , a1 + a2 + a3 = b1 + b2 + b3 .
Prueba. De las hipótesis se observa que hay dos casos para analizar:
Caso 1:
b1 ≥ a2 ⇔ a1 + b1 ≥ a1 + a2
⇔ a1 ≥ a1 + a2 − b1
X X
xa1 y a2 z a3 ≥ z a3 xa1 +a2 −b1 y b1 + xb1 y a1 +a2 −b1
sim cı́c
X
= xb1 y a1 +a2 −b1 z a3 + xb1 y a3 z a1 +a2 −b1
cı́c
X
= xb1 y a1 +a2 −b1 z a3 + y a3 z a1 +a2 −b1
cı́c
X
≥ xb1 y b2 z b3 + y b3 z b2
cı́c
X
= xb1 y b2 z b3 + xb1 y b3 z b2
cı́c
X
= xb1 y b2 z b3
sim
Caso 2: Si b1 ≥ a2
⇒ 3b1 ≥ b1 + b2 + b3 = a1 + a2 + a3 ≥ b1 + a2 + a3
⇒ 3b1 ≥ b1 + a2 + a3
⇒ 2b1 ≥ a2 + a3
⇒ b1 ≥ a2 + a3 − b1
además a1 ≥ a2 ≥ b1 ⇒ a1 ≥ a2 + a3 − b1 .
Luego tenemos que máx {a2 , a3 } ≥ máx {b1 , a2 + a3 − b1 } y máx {a1 , a2 + a3 − b1 } ≥
máx {b2 , b3 }.
En efecto:
X X
xa1 y a2 z a3 = (xa1 y a2 z a3 + xa1 z a2 y a3 )
sim cı́c
X
= xa1 (y a2 z a3 + z a2 y a3 )
cı́c
126 10. Desigualdades Simétricas y Cı́clicas
X
≥ xa1 y b1 z a2 +a3 −b1 + y a2 +a3 −b1 z b1
cı́c
X
= xa1 y b1 z a2 +a3 −b1 + xa1 y a2 +a3 −b1 z b1
cı́c
X
= xa1 y b1 z a2 +a3 −b1 + xa2 +a3 −b1 y b1 z a1
cı́c
X
= y b1 xa1 z a2 +a3 −b1 + xa2 +a3 −b1 z a1
cı́c
X
≥ y b1 xb2 z b3 + xb3 z b2
cı́c
X
= xb1 y b2 z b3
sim
2
Aplicación 10.3
Pruebe para todo a, b, c números reales positivos la desigualdad
a b c 3
+ + ≥ .
b+c a+c a+b 2
Prueba. La desigualdad a demostrar es equivalente a
2 [a(a + c)(a + b) + b(b + c)(a + b) + c(b + c)(a + c)] ≥ 3(b + c)(a + c)(a + b),
efectuando
Aplicación 10.4
(IMO 1995). Sean a, b, c números reales positivos tales que abc = 1, pruebe que
1 1 1 3
+ 3 + 3 ≥ .
a3 (b + c) b (c + a) c (a + b) 2
Prueba. Como los denominadores del primer miembro son de grado 4 y abc = 1,
entonces la desigualdad es equivalente a
1 1 1 3
+ 3 + 3 ≥ ,
a3 (b + c) b (c + a) c (a + b) 2(abc)4/3
a = x3 , b = y 3 , c = z 3 , con x, y, z > 0,
luego tenemos:
1 1 1 3 3x5 y 5 z 5
+ + ≥ =
x9 (y 3 + z 3 ) y 9(z 3 + x3 ) z 9 (x3 + y 3) 2(xyz)4 2(x9 y 9 z 9 )
⇔ 2 [y 9z 9 (z 3 + x3 )(x3 + y 3 ) + x9 z 9 (y 3 + z 3 )(x3 + y 3) + x9 z 9 (y 3 + z 3 )(z 3 + x3 )]
≥ 3 [x5 y 5z 5 (y 3 + z 3 )(z 3 + x3 )(x3 + y 3 )] ,
efectuando
X X X X
x12 y 12 + 2 x12 y 9 z 3 + x9 y 9 z 6 ≥ 3 x11 y 8z 5 + 6x8 y 8 z 8
sim sim sim sim
X X X X
⇔ x12 y 12 − x11 y 8 z 5 + 2 x12 y 9z 3 − x11 y 8z 5
sim sim sim sim
X X
+ x9 y 9 z 6 − x8 y 8 z 8 ≥ 0.
sim sim
(i) α1 ≥ α2 ≥ · · · ≥ αn y β1 ≥ β2 ≥ · · · ≥ βn .
(ii) α1 + α2 + · · · + αk ≥ β1 + β2 + · · · + βk ; k = 1, 2, . . . , (n − 1).
(iii) α1 + α2 + · · · + αn = β1 + β2 + · · · + βn .
3 3 √ √ √
(x1 + x32 + x33 + x34 ) ≥ x1 x4 x2 x3 + x2 x4 x1 x3 + x3 x4 x1 x2
2
√ √ √
+x2 x3 x1 x4 + x1 x2 x3 x4 + x1 x3 x2 x4 .
X X
Prueba. Como x31 = x31 · (x02 x03 x04 ), entonces tenemos 6 permutaciones
sim √sim
para (x2 , x3 , x4 ) y para x1 x2 x3 x4 , tenemos 2 permutaciones para x1 x2 y 2 per-
√
mutaciones para x3 x4 , en total 4; luego, la desigualdad se puede escribir equiva-
lentemente como
√ √ √
6(x31 + x32 + x33 + x34 ) ≥ 4[x1 x4 x2 x3 + x2 x4 x1 x3 + x3 x4 x1 x2
√ √ √
+x2 x3 x1 x4 + x1 x2 x3 x4 + x1 x3 x2 x4 ].
X X √
⇔ x31 ≥ x1 x2 x3 x4 ,
sim sim
1. Sean x, y números reales positivos tales que a y b son las medias aritmética y
geométrica respectivamente, pruebe que
2. Pruebe que
2
1 1 2
1+ 1+ ≥ 1+
x y x+y
3. Sean x, y, z números reales positivos, pruebe que al menos una de las siguientes
desigualdades es verdadera:
xy ≥ 0
yz ≥ 0
zx ≥ 0.
129
130 11. Problemas
6. Pruebe que
p √ p
m2 + y 2 + n2 + x2 ≥ (m + n)2 + (x + y)2
1 + ab 1 + bc 1 + ac
+ + ≥ 3.
1+a 1+b 1+c
a b c 3
+ + ≥ .
b+c c+a a+b 2
(Desigualdad de Nesbit).
9. Pruebe que si x, y, z son números reales positivos y x + y + z = 1, entonces
1 1 1
1+ 1+ 1+ ≥ 64.
x y z
c3 d 3
+ ≥ 1.
a b
a b c
+ + ≥ 1.
b + 2c c + 2a a + 2b
x2 y2
+ ≥ 8,
y−1 x−1
16. Sean a, b, c números reales positivos tales que ab + bc + ca ≤ 3abc, pruebe que
a + b + c ≤ a3 + b3 + c3 .
1 1 1
+ + ≥ 1.
1 + 2ab 1 + 2bc 1 + 2ca
1 1 1 3
+ + ≥ .
1 + ab 1 + bc 1 + ca 2
1 1 1
√ +√ +√ ≥ 1.
1+a3 1+b3 1 + c3
132 11. Problemas
1 1 1
2
+ 2
≥ .
(1 + a) (1 + b) 1 + ab
22. Sean a, b, c, d números reales positivos tales que abcd = 1, pruebe que
1 1 1 1
2
+ 2
+ 2
+ ≥ 1.
(1 + a) (1 + b) (1 + c) (1 + d)2
24. (IMO 1995). Sean a, b, c números reales positivos tales que abc = 1, pruebe
que
1 1 1 3
+ 3 + 3 ≥ .
a3 (b + c) b (c + a) c (a + b) 2
26. Sean x, y, z números reales que pertenecen al intervalo [−1, 1] tales que x +
y + z = 0, pruebe que
p p √
1 + x + y 2 + 1 + y + z 2 + 1 + z + x2 ≥ 3.
a2 (b + c − a) + b2 (a + c − b) + c2 (a + b − c) ≤ 3abc.
a b c d
+ 2 + 2 + 2 ≥ 2.
b2 2
+c +d 2 2
c +d +a2 2
d +a +b2 a + b2 + c2
32. Sean a, b, c, d números reales no negativos tales que a2 +b2 +c2 +d2 = 4, pruebe
que
a3 + b3 + c3 + d3 ≤ 8.
a b c
+ + ≥ 1.
3a − b + c 3b − c + a 3c − a + b
34. Pruebe que si a, b, c son números reales positivos tales que abc = 1,entonces
1 1 1 1 1 1
+ + ≤ + + .
1+a+b 1+b+c 1+c+a 2+a 2+b 2+c
35. (IMO 2005). Dados a, b, c números reales positivos que satisfacen abc ≥ 1,
pruebe que
a5 − a2 b5 − b2 c5 − c2
+ + ≥ 0.
a5 + b2 + c2 b5 + c2 + a2 c5 + a2 + b2
134 11. Problemas
√ √ √
a+ b+ c ≥ ab + bc + ca.
x y z 2(x + y + z)
1+ 1+ 1+ ≥2+ √ .
y z x 3 xyz
1 1 1 1
+ 3 + 3 ≤ .
a3 b3 3 3
+ + 3abc b + c + 3abc c + a + 3abc abc
a b c 3
+ + ≥ .
1 + b2 1 + c2 1 + a2 2
a3 b3 c3 d3 a+b+c+d
+ + + ≥ .
a2 + b2 b2 + c2 c2 + d2 d2 + a2 2
41. Sean a, b, c, d números reales positivos tales que abcd = 1, pruebe que
1 1 1
S = + +
1 + ab + bc + ca 1 + bc + cd + ab 1 + cd + da + ac
1
+ ≤ 1.
1 + da + ab + bd
1 1 1
+ + ≥ 1.
2ab2 +1 2bc2 2
+ 1 2ca + 1
135
a2 (b + c − a) b2 (c + a − b) c2 (a + b − c) ab + bc + ca
+ + ≤ .
b+c c+a a+b 2
√ √ √
a2 − bc b + c + b2 − ca c + a + c2 − ab a + b ≥ 0.
2 2 2
a b c 3
+ + ≥ .
a+b b+c c+a 4
1 1 1 1
+ + + = 1,
1+a 1+b 1+c 1+d
a b c
+ + ≥ 1.
3a − b + c 3b − c + a 3c − a + b
a b c d
A= √
3
+√
3
+√
3
+√
3
≥ 1.
a3 + 63bcd b3 + 63acd c3 + 63bda d3 + 63abc
55. Dados a, b, c números reales tales que a, b, c ∈ [0; 1], pruebe que
a b c
+ + + (1 − a)(1 − b)(1 − c) ≤ 1.
b+c+1 c+a+1 a+b+1
a b c 3
+ + ≤ .
2a + b + c 2b + c + a 2c + a + b 4
1 1 1
+ 2 + 2 ≤ 1.
a2 +2 b +2 c +2
a3 b3 c3 3 2 1
+ + ≥ a + b2 + c2 − (ab + bc + ca).
a+b b+c c+a 4 4
138 11. Problemas
ab bc ca 1
S= + + ≤ (a + b + c).
2c + a + b 2a + b + c 2b + c + a 4
65. Si a ≥ b ≥ c ≥ d ≥ e, entonces
x y 2
√ +p ≥√ .
1−x2 1 − y2 3
√
x4 + y 4 + z 2 ≥ 8xyz.
139
pruebe que
xy + yz + zx ≥ a2 + b2 + c2 .
n X
X n
i · j · cos (ai − aj ) ≥ 0.
i=1 j=1
pruebe que
xy + yz + zx ≥ (a + b + c)2 .
a b c d e f
+ + + + + ≥ 3.
b+c c+d d+e e+f f +a a+b
140 11. Problemas
a3 + b3 + c3 x3 + y 3 + z 3 t3 + u3 + v 3 ≥ (axt + byu + czv)3 .
a b c
√ +√ +√ ≥ 1.
a2 + 8bc 2
b + 8ca 2
c + 8ab
1 1 1
79. (APMO 2002). Sean a, b y c números reales positivos que satisfacen + + =
a b c
1, pruebe que
√ √ √ √ √ √ √
a + bc + b + ca + c + ab ≥ abc + a+ b+ c.
80. Sean a, b, c números reales positivos que satisfacen abc = 1, pruebe que
3 6
1+ ≥ .
a+b+c ab + ac + bc
x+y+z xy + yz + zx
√ ≥p p √ .
3 3 x2 + xy + y 2 + y 2 + yz + z 2 + z 2 + zx + x2
84. Dados a, b, c números reales tales que a2 +b2 +c2 ≤ 8, pruebe que ab+bc+2ca ≥
−8.
141
a b c
+ + ≥ 3.
b+c−a c+a−b a+b−c
S S S n2
+ +···+ ≥ .
S − a1 S − a2 S − an n−1
1 1 1
+ +···+ = 1,
1 + x1 1 + x2 1 + xn
142 11. Problemas
pruebe que
x1 x2 · · · xn ≥ (n − 1)n .
ab bc ca
+ + ≥ 3.
c a b
a+b+c
aa · bb · cc ≥ (abc) 3 .
a3 b3 c3 3(ab + ac + bc)
2 2
+ 2 2
+ 2 2
≥ .
b − bc + b c − ac + a a − ab + b a+b+c
3 1 − a + a2 1 − b + b2 1 − c + c2 ≥ 1 + abc + a2 b2 c2 .
143
101. Sean a, b, c números reales positivos tales que abc ≥ 1, pruebe que
a3 + b3 + c3 ≥ ab + ac + bc.
√ √ √ √ √ √
ab + c + bc + a + ca + b ≥ 1 + ab + bc + ca.
103. Sean a, b números reales positivos tales que a + b ≤ 1, halle el mı́nimo valor
de f (a, b) = ab + (ab)−1 .
3
104. Sean a, b, c números reales positivos tales que a + b + c ≤ , halle el mı́nimo
r r r 2
1 1 1
de f (a, b, c) = a2 + 2 + b2 + 2 + c2 + 2 .
b c a
√ √ √
bc a − 2 + ca 3 b − 6 + ab 4 c − 12
f (a, b, c) = .
abc
a3 b3 c3
106. Sean a, b, c números reales positivos, pruebe que + + ≥ ab + bc + ca.
b c a
107. Sean a, b, c, d números reales positivos, halle el mı́nimo valor de la función
a b c d
f (a, b, c, d) = + + +
b+c+d c+d+a d+a+b a+b+c
b+c+d c+d+a d+a+b a+b+c
+ + + + .
a b c d
144 11. Problemas
a3 (b + c + d) + b3 (c + d + a) + c3 (d + a + b) + d3 (a + b + c)
a+b+c √
3 √ √ 2 √ √ 2 √ √ 2
− abc ≤ máx a− b , b− c , c− a .
3
3
3 3 b+c
3
a +b +c +2 a− ≥ 3abc.
2
a2 b2 c2
+ + ≥ 2.
(b − c)2 (c − a)2 (a − b)2
4x2
√ 2 < 2x + 9.
1 − 1 + 2x
a2 (b + c − a) + b2 (a + c − b) + c2 (a + b − c) ≤ 3abc.
145
x1 x2 xn 1 1 1
2
+ 2 +···+ 2 ≥ + +···+ .
1 2 n 1 2 n
n+1
|x1 + x2 + · · · + xn | = 1 y |xi | ≤ , para todo i = 1, 2, . . . , n,
2
n+1
|y1 + 2y2 + · · · + nyn | ≤ .
2
pruebe que ti , tj , tk son los lados de un triángulo para todo i, j, k con 1 ≤ i <
j < k ≤ n.
119. (IMO 2000). Sean a, b, c números reales positivos tales que abc = 1, pruebe
que
1 1 1
a−1+ b−1+ c−1+ ≤ 1.
b c a
146 11. Problemas
n
X nX
|i − j| |xi − xj | = |xi − xj |
i,j=1
2 i,j
si x1 ≤ x2 ≤ · · · ≤ xn .
n
X n2 (n2 − 1)
(i − j)2 = .
i,j=1
6
n
!2 n
X 2 2 X
|xi − xj | ≤ n −1 (xi − xj )2 .
i,j=1
3 i,j=1
ab(a2 − b2 ) + bc(b2 − c2 ) + ca(c2 − a2 ) ≤ M a2 + b2 + c2 2
125. Sean a, b, c, d, e números reales positivos tales que abcde = 1, pruebe que
x2 x2 x2
+ + ≥1 (11.1)
(x − 1)2 (x − 1)2 (x − 1)2
1. Sean x, y números reales positivos tales que a y b son las medias aritmética y
geométrica respectivamente, pruebe que
x+y √
Prueba. Sean a = , b = xy, y como se cumple:
2
√
2 xy ≤ x + y
√
⇔ 1 + 2 xy + xy ≤ 1 + x + y + xy
√
⇔ (1 + xy)2 ≤ (1 + x)(1 + y)
⇔ (1 + b)2 ≤ (1 + x)(1 + y). (12.1)
También se cumple
4xy ≤ (x + y)2
(x + y)2
⇔ xy ≤
4
(x + y)2
⇔ 1 + x + y + xy ≤ 1 + x + y +
4
149
150 12. Soluciones
2
x+y
⇔ (1 + x)(1 + y) ≤ 1 +
2
⇔ (1 + x)(1 + y) ≤ (1 + a)2 (12.2)
2. Pruebe que
2
1 1 2
1+ 1+ ≥ 1+
x y x+y
1 1 1 4 4
1+ + + ≥ 1+ +
x y xy x + y (x + y)2
⇔ (x + y + 1)(x + y)2 ≥ 4(x + y)xy + 4xy
⇔ (x + y + 1)(x + y)2 ≥ 4xy(x + y + 1)
⇔ (x + y)2 ≥ 4xy
⇔ (x − y)2 ≥ 0.
2
3. Sean x, y, z números reales positivos, pruebe que al menos una de las siguientes
desigualdades es verdadera:
xy ≥ 0
yz ≥ 0
zx ≥ 0.
151
xy < 0
yz < 0
zx < 0.
Luego tendrı́amos que −xy > 0, −yz > 0, −zx > 0, y multiplicando −x2 y 2 z 2 >
0, lo cual es una contradicción. Por lo tanto una de las desigualdades debe ser
verdadera. 2
√ √
de donde − 13 ≤ 2a + 3b ≤ 13. 2
Solución. Tomamos las ternas (2, 3, 4) y (a, b, c), luego aplicamos Cauchy-
Schwarz:
6. Pruebe que
p √ p
m2 + y 2 + n2 + x2 ≥ (m + n)2 + (x + y)2
+(x2 + y 2 + 2xy)
p √ 2
⇔ m2 + y2 + n2 + x2 ≥ (m + n)2 + (x + y)2
p √ p
⇔ m2 + y 2 + n2 + x2 ≥ (m + n)2 + (x + y)2.
1 + ab 1 + bc 1 + ac
+ + ≥ 3.
1+a 1+b 1+c
Prueba.
1 + ab c + abc c+1
= = ; similarmente
1+a c(1 + a) c(1 + a)
1 + bc a+1
=
1+b a(1 + b)
1 + ac b+1
=
1+c b(1 + c)
153
1
vemos que al multiplicar las tres últimas fracciones obtenemos = 1, en-
abc
tonces aplicando MA ≥ MG, tenemos:
entonces
1 + ab 1 + bc 1 + ac
∴ + + ≥ 3.
1+a 1+b 1+c
2
a b c 3
+ + ≥ .
b+c c+a a+b 2
(Desigualdad de Nesbit).
1 1 1
⇔ 2(a + b + c) + + ≥9
b+c c+a a+b
1 1 1
⇔ ((b + c) + (c + a) + (a + b)) + + ≥9
b+c c+a a+b
(b + c) + (c + a) + (a + b) 3
⇔ ≥
3 1 1 1
+ +
b+c c+a a+b
aplicando MA ≥ MG:
x+x+y+z √
≥ 4x·x·y·z
4
p
2x + y + z ≥ 4 4 x2 yz; similarmente
p
x + 2y + z ≥ 4 4 xy 2 z
p
x + y + 2z ≥ 4 4 xyz 2
2
155
c3 d 3
+ ≥ 1.
a b
2 2
c3 d 3 c4 d4 (c2 ) (d2 )
+ = + = +
a b ac bd ac bd
luego, utilizando el Lema de Titu, tenemos
2 !2 !2
2 2 2
c3 d 3 (c2 ) (d2 ) (c2 + d2 )
+ = + ≥
a b ac bd ac + bd
4
(c2 + d2 )
=
(ac + bd)2
3
(c2 + d2 ) (c2 + d2 )
=
(ac + bd)2
(a2 + b2 ) (c2 + d2 )
=
(ac + bd)2
entonces
2
c3 d 3 (a2 + b2 ) (c2 + d2 )
+ ≥ ≥1
a b (ac + bd)2
a b c
+ + ≥ 1.
b + 2c c + 2a a + 2b
156 12. Soluciones
a b c a2 b2 c2
+ + = + +
b + 2c c + 2a a + 2b ab + 2ac bc + 2ab ac + 2bc
a2 b2 c2 (a + b + c)2
+ + ≥ ,
ab + 2ac bc + 2ab ac + 2bc 3(ab + ac + bc)
(a + b + c)2
≥ 1
3(ab + ac + bc)
⇔ a2 + b2 + c2 + 2(ab + bc + ca) ≥ 3(ab + ac + bc)
⇔ a2 + b2 + c2 ≥ ab + ac + bc,
a b c
∴ + + ≥ 1.
b + 2c c + 2a a + 2b
2
x2 y2
+ ≥ 8,
y−1 x−1
x2 − 4x + 4 ≥ 0
⇔ x2 ≥ 4(x − 1)
√
⇔ x≥2 x−1
x
⇔ √ ≥2
x−1
157
similarmente
y
√ ≥2
y−1
x y
como √ ,√ ∈ R+ ; entonces aplicando MA ≥ MG
x−1 y−1
x2 y2 s
+
y−1 x−1 x2 y2
≥ ·
2 y−1 x−1
x2 y2 xy x y
+ ≥ 2· √ √ = 2· √ ·√
y−1 x−1 x−1· y−1 x−1 y−1
≥ 2(2)(2) = 8
x2 y2
∴ + ≥ 8.
y−1 x−1
2
Pero
1 1 1
[(a + c) + (b + a) + (c + b)] + + ≥9
a+c b+a c+b
1 1 1
⇔ 2(a + b + c) + + ≥9
a+c b+a c+b
1 1 1
⇔ 4 + + ≥9
a+c b+a c+b
1 1 1 9
⇔ + + ≥
a+c b+a c+b 4
Prueba. 2
a b + ab2 = |ab(a + b)| = |ab||a + b|
pero
√ √ √
|a + b| ≤ 2 a2 + b2 = 2 · 1 = 2
√
|a + b| ≤ 2 (12.3)
además
a2 + b2 ≥ 2|a||b|
1
⇔ ≥ |ab|
2
1
⇔ |ab| ≤ (12.4)
2
159
√
2
∴ a2 b + ab2 ≤ .
2
2
(a + b)2 = a2 + 2ab + b2
√
3
= a2 + ab + ab + b2 ≥ a2 + 3 a2 b4
⇒ (a + b)2 ≥ a2 + 3a2/3 · b4/3
= a2/3 a4/3 + 3b4/3
p √
⇒ (a + b) ≥ a2/3 (a4/3 + 3b4/3 ) = a1/3 a4/3 + 3b4/3
sumando obtenemos
√ √
x y
√ +√ ≥ 1.
x + 3y y + 3x
2
160 12. Soluciones
16. Sean a, b, c números reales positivos tales que ab + bc + ca ≤ 3abc, pruebe que
a + b + c ≤ a3 + b3 + c3 .
ab + bc + ca
⇒ ≤ 3
abc
1 1 1
⇒ + + ≤ 3
a b c
1 1 1
Pero (a + b + c) + + ≥9
a b c
⇒ a+b+c≥3
además
3(a + b + c) ≤ (a + b + c)2
2
= a3/2 · a−1/2 + b3/2 · b−1/2 + c3/2 · c−1/2
≤ (a3 + b3 + c3 ) (a−1 + b−1 + c−1 )
≤ (a3 + b3 + c3 ) 3
⇒ 3(a + b + c) ≤ 3 a3 + b3 + c3
∴ a + b + c ≤ a3 + b3 + c3 .
1 1 1
+ + ≥ 1.
1 + 2ab 1 + 2bc 1 + 2ca
161
a2 + b2 ≥ 2ab
⇔ 1 + a2 + b2 ≥ 1 + 2ab
1 1
⇔ ≥
1 + 2ab 1 + a2 + b2
en efecto
1 1 1 12 12 12
+ + ≥ + +
1 + 2ab 1 + 2bc 1 + 2ca 1 + a2 + b2 1 + b2 + c2 1 + a2 + c2
12 12 12 (1 + 1 + 1)2
+ + ≥
1 + a2 + b2 1 + b2 + c2 1 + a2 + c2 3 + 2(a2 + b2 + c2 )
32
= =1
3 + 2(3)
1 1 1
∴ + + ≥ 1.
1 + 2ab 1 + 2bc 1 + 2ca
2
1 1 1 3
+ + ≥ .
1 + ab 1 + bc 1 + ca 2
Prueba.
2
b2 + c}2 ≥ ab + ac + bc
|a + {z
3
⇔ 3 ≥ ab + ac + bc
⇔ 6 ≥ (1 + ab) + (1 + ac) + (1 + bc)
9 9
⇔ ≤
6 3 + ab + ac + bc
162 12. Soluciones
9 3
⇔ ≥
3 + ab + ac + bc 2
1 1 1
[(1 + ab) + (1 + ac) + (1 + bc)] + + ≥9
1 + ab 1 + ac 1 + bc
1 1 1 9
⇔ + + ≥
1 + ab 1 + ac 1 + bc 3 + ab + ac + bc
1 1 1 3
∴ + + ≥ .
1 + ab 1 + bc 1 + ca 2
2
Prueba.
2
(a + b + c)4 = ((a + b + c)2 )
2
2
= a b2 + c}2 + 2(ab + ac + bc)
| + {z | {z }
x y
2
= (x + y)
≥ 4xy
= 4 (a2 + b2 + c2 ) (2)(ab + ac + bc)
= 8 (a2 + b2 + c2 ) (ab + ac + bc)
∴ (a + b + c)4 ≥ 8 a2 + b2 + c2 (ab + ac + bc).
2
163
1 1 1
√ +√ +√ ≥ 1.
1+a3 1+b3 1 + c3
1 + a3 = (1 + a)(1 − a + a2 ),
entonces
(1 + a) + (1 − a + a2 ) p
≥ (1 + a)(1 − a + a2 )
2
2 + a2 √
⇔ ≥ 1 + a3
2
2 1
⇒ 2
≤√ ;
2+a 1 + a3
luego
1 1 1 2 2 2
√ +√ +√ ≥ 2
+ 2
+
1+a3 1+b3 1+c 3 2+a 2+b 2 + c2
2 2 2
2
+ 2
+ ≥1
2+a 2+b 2 + c2
1 1 1 1
⇔ 2
+ 2
+ 2
≥
2+a 2+b 2+c 2
⇔ (2 + b2 ) (2 + c2 ) + (2 + a2 ) (2 + c2 ) + (2 + a2 ) (2 + b2 )
1
≥ (2 + a2 ) (2 + b2 ) (2 + c2 )
2
164 12. Soluciones
⇔ a2 + b2 + c2 ≥ 12.
Es verdadera pues
√
3
a2 + b2 + c2 ≥ 3 a2 b2 c2 = 12.
1 1 1
2
+ 2
≥ .
(1 + a) (1 + b) 1 + ab
Prueba.
1 1 1
2
+ 2
≥ ;
(1 + a) (1 + b) 1 + ab
⇔ (2 + 2a + 2b + a2 + b2 ) (1 + ab) ≥ (1 + 2a + a2 )(1 + 2b + b2 )
⇔ a3 b + ab3 + 1 ≥ 2ab + a2 b2
22. Sean a, b, c, d números reales positivos tales que abcd = 1, pruebe que
1 1 1 1
+ + + ≥ 1.
(1 + a)2 (1 + b)2 (1 + c)2 (1 + d)2
1 1 1 1 1 1
2
+ 2
+ 2
+ 2
≥ +
(1 + a) (1 + b) (1 + c) (1 + d) 1 + ab 1 + cd
| {z } | {z }
1 ab
= +
1 + ab ab + abcd
1 ab
= + = 1.
1 + ab ab + 1
1 1 1 1
∴ 2
+ 2
+ 2
+ ≥ 1.
(1 + a) (1 + b) (1 + c) (1 + d)2
24. (IMO 1995). Sean a, b, c números reales positivos tales que abc = 1, pruebe
que
1 1 1 3
+ 3 + 3 ≥ .
a3 (b+ c) b (c + a) c (a + b) 2
x2 y2 z2 (x + y + z)2
+ + ≥
y+z x+z x+y (y + z) + (x + z) + (x + y)
(x + y + z)2 x+y+z 3 √ 3
= = ≥ · 3 xyz = .
2(x + y + z) 2 2 | {z } 2
1
2
167
2 1 1 2 1 1 2 1 1
a + +b + +c + ≥ 2(a + b + c)
b c a c a b
Prueba. Como
a2 + b2 ≥ 2ab
⇔ a2 − ab + b2 ≥ ab; multiplicando por(a + b)
⇔ (a + b) (a2 − ab + b2 ) ≥ ab(a + b)
⇔ a3 + b3 ≥ ab(a + b)
a3 b3
⇔ + ≥a+b
ab ab
a2 b2
⇔ + ≥ a + b; similarmente
b a
a2 c2
+ ≥a+c
c a
b2 c2
+ ≥b+c
c b
2 1 1 2 1 1 2 1 1
a + +b + +c + ≥ 2(a + b + c)
b c a c a b
26. Sean x, y, z números reales que pertenecen al intervalo [−1, 1] tales que x +
y + z = 0, pruebe que
p p √
1 + x + y2 + 1 + y + z2 + 1 + z + x2 ≥ 3.
168 12. Soluciones
√ √ √
Prueba. Si ab ≥ 0, veamos que 1+a+ 1+b≥1+ 1 + a + b.
En efecto
√ √ √
1+a+ 1+b≥ 1+ 1+a+b
p √
⇔ 2 + a + b + 2 (1 + a)(1 + b) ≥ 1 + 2 1 + a + b + 1 + a + b
√ √
⇔ 1 + a + b + ab ≥ 1+a+b
⇔ 1 + a + b + ab ≥ 1 + a + b
⇔ ab ≥ 0.
elevando al cuadrado
⇔ 1 + z 3 ≥ 1 − 2z 2 + z 4
⇔ z 3 ≥ z 4 − 2z 2
⇔ z 4 − z 3 − 2z 2 ≤ 0
⇔ z 2 (z 2 − z − 2) ≤ 0
⇔ z 2 (z − 2)(z + 1) ≤ 0
⇔ z 2 (2 − z)(z + 1) ≥ 0
⇒ a ≥ b; a ≥ c.
elevando al cuadrado
x
Prueba. Definimos la función f (x) = √ ; x < 1, entonces
1−x
2−x
f ′ (x) = √
2(1 − x) 1 − x
√
′′ 1 − x(4 − x)
f (x) = >0
4(1 − x)3
reemplazando x + y + z = 1, tenemos
1
1 x y z
r 3 ≤ √ +√ +√
1 3 1−x 1−y 1−z
1−
3
171
r
x y z 3
∴ √ +√ +√ ≥ .
y+z z+x x+y 2
1
La igualad ocurre si y sólo si x = y = z = . 2
3
a2 (b + c − a) + b2 (a + c − b) + c2 (a + b − c) ≤ 3abc.
a2 (b + c) − a3 + b2 (a + c) − b3 + c2 (a + b) − c3 ≤ 3abc
⇔ a2 (b + c) + b2 (a + c) + c2 (a + b) ≤ a3 + b3 + c3 + 3abc
⇔ 0 ≤ (a3 − a2 (b + c) + abc) + (b3 − b2 (a + c) + abc) + (c3 − c2 (a + b) + abc)
⇔ 0 ≤ a (a2 − a(b + c) + bc) + b (b2 − b(a + c) + ac) + c (c2 − c(a + b) + ab)
⇔ 0 ≤ a(a − b)(a − c) + b(b − a)(b − c) + c(c − a)(c − b)
⇔ a(a − b)(a − c) + b(b − a)(b − c) + c(c − a)(c − b) ≥ 0
máx {a; b} = a
Definamos la función
f (a, b) = ab + a = a(b + 1)
⇒ f 2 (a, b) = a2 (b + 1)2 = (1 − b2 ) (1 + b)2
= (1 − b)(1 + b)3
1
= (3 − 3b)(1 + b)3
3
aplicando MG ≤ MG
4
1 3 1 3 − 3b + 1 + b + 1 + b + 1 + b
(3 − 3b)(1 + b) ≤
3 3 4
4
1 3 27
= =
3 2 16
27
⇒ f 2 (a, b) ≤ .
16
√
3 3
∴ f (a, b) ≤ .
4
2
a b c d
+ 2 + 2 + 2 ≥ 2.
b2 2
+c +d 2 2
c +d +a2 2
d +a +b2 a + b2 + c2
√ 1+x
Prueba. Recordar que x≤ , entonces tenemos que
2
r b2 + c2 + d2
2
b +c +d 2 2 1+ a2 + b2 + c2 + d2
≤ a2 =
a2 2 2a2
173
a 2a2
⇒ ≥ ; similarmente
b + c2 + d 2
2 a2 + b2 + c2 + d2
b 2b2
≥ 2
c + d2 + a2
2 a + b2 + c2 + d2
c 2c2
≥ 2
d + a2 + b2
2 a + b2 + c2 + d2
d 2d2
≥ 2
a + b2 + c2
2 a + b2 + c2 + d2
a b c d
+ 2 + 2 + 2 ≥ 2.
b2 2
+c +d 2 2
c +d +a2 2
d +a +b2 a + b2 + c2
2
32. Sean a, b, c, d números reales no negativos tales que a2 +b2 +c2 +d2 = 4, pruebe
que
a3 + b3 + c3 + d3 ≤ 8.
de donde a3 + b3 + c3 + d3 ≥ 1
como vemos no hemos acotado superiormente, es decir esta propiedad no ayuda
resolver el problema; buscaremos otra opción.
De la condición a2 + b2 + c2 + d2 = 4, se deduce que
a2 ≤ 4;
⇒ a ≤ 2; a ≥ 0
174 12. Soluciones
⇒ a3 ≤ 2a2 ; similarmente
b3 ≤ 2b2
c3 ≤ 2c2
d3 ≤ 2d2
a3 + b3 + c3 + d3 ≤ 2 (a2 + b2 + c2 + d2 )
a3 + b3 + c3 + d3 ≤ 2 (4)
de donde a3 + b3 + c3 + d3 ≤ 8.
La igualdad ocurre si y sólo si (a, b, c, d) = (2, 0, 0, 0), y sus respectivas per-
mutaciones. [8, pág 15] 2
a b c
+ + ≥ 1.
3a − b + c 3b − c + a 3c − a + b
a b c
+ + ≥1
3a − b + c 3b − c + a 3c − a + b
4a 4b 4c
⇔ + + ≥4
3a − b + c 3b − c + a 3c − a + b
4a 4b 4c
⇔ −1 + −1 + −1 ≥1
3a − b + c 3b − c + a 3c − a + b
a+b−c b+c−a c+a−b
⇔ + + ≥1
3a − b + c 3b − c + a 3c − a + b
175
(a + b + c)2
= = 1.
(a + b + c)2
34. Pruebe que si a, b, c son números reales positivos tales que abc = 1,entonces
1 1 1 1 1 1
+ + ≤ + + . (12.7)
1+a+b 1+b+c 1+c+a 2+a 2+b 2+c
x=a+b+c≥3
x ≥ 3; y ≥ 3
x2 ≥ 3y
Reemplazando tenemos
x2 + 4x + y + 3 12 + 4x + y
2
≤
x + 2x + y + xy 9 + 4x + 2y
x2 + 4x + y + 3 12 + 4x + y
⇔ 2
−1≤ −1
x + 2x + y + xy 9 + 4x + 2y
2x − xy + 3 3−y
⇔ ≤ −1
x2 + 2x + y + xy 9 + 4x + 2y
⇔ 5x2 + 24x + y 2 + 3y + 27 ≤ 6xy + 3x2 y + xy 2
5 1
⇔ 5x2 + 24x + y 2 + 3y + 27 ≤ 5xy + xy + x2 y + x2 y + x2 y + xy 2
3 3
5 1
5x2 ≤ x2 y; y 2 ≤ x2 y; 9x ≤ xy 2
3 3
15x ≤ 5xy; 3y ≤ xy; 27 ≤ x2 y.
35. (IMO 2005). Dados a, b, c números reales positivos que satisfacen abc ≥ 1,
pruebe que
a5 − a2 b5 − b2 c5 − c2
+ + ≥ 0.
a5 + b2 + c2 b5 + c2 + a2 c5 + a2 + b2
177
a5 − a2 b5 − b2 c5 − c2
−1 + 5 −1 + 5 − 1 ≥ −3
a5 + b2 + c2 b + c2 + a2 c + a2 + b2
(a2 + b2 + c2 ) (a2 + b2 + c2 ) (a2 + b2 + c2 )
⇔ − − 5 − 5 ≥ −3
a5 + b2 + c2 b + c2 + a2 c + a2 + b2
(a2 + b2 + c2 ) (a2 + b2 + c2 ) (a2 + b2 + c2 )
⇔ + 5 + 5 ≤3
a5 + b2 + c2 b + c2 + a2 c + a2 + b2
de donde
2
(a5 + b2 + c2 ) (bc + b2 + c2 ) ≥ (a2 + b2 + c2 )
bc + b2 + c2 a2 + b2 + c2
⇔ ≥
a2 + b2 + c2 a5 + b2 + c2
a2 + b2 + c2 bc + b2 + c2
⇔ 5 ≤ 2 ; similarmente
a + b2 + c2 a + b2 + c2
a2 + b2 + c2 ac + a2 + c2
≤ 2 ;
b5 + c2 + a2 a + b2 + c2
a2 + b2 + c2 ab + a2 + b2
≤ 2
c5 + a2 + b2 a + b2 + c2
a2 + b2 + c2 a2 + b2 + c2 a2 + b2 + c2
+ +
a5 + b2 + c2 b5 + c2 + a2 c5 + a2 + b2
bc + ac + ab + 2 (a2 + b2 + c2 ) 3 (a2 + b2 + c2 )
≤ ≤ = 3.
a2 + b2 + c2 a2 + b2 + c2
178 12. Soluciones
Recordar que
ab + ac + bc ≤ a2 + b2 + c2 .
√ √ √
a+ b+ c ≥ ab + bc + ca.
√ √ √
a2 + b2 + c2 + 2( a + b + c) ≥ (a b + }c)2 = 9
| +{z
3
√ 2 √ √
a + 2 a + b + 2 b + c2 + 2 c ≥ 9.
2
Pero
√ √ √ p √ √
a2 + 2 a = a2 + a + a ≥ 3 3 a2 a a = 3a
entonces
√
a2 + 2 a ≥ 3a; similarmente
√
b2 + 2 b ≥ 3b
√
c2 + 2 c ≥ 3c
x y z x z y 2(x + y + z)
1+ + + + + + +1≥2+ √
y z x z y x 3 xyz
simplificando
x y z x z y 2(x + y + z)
+ + + + + ≥ √
y z x z y x 3 xyz
Aplicando MA ≥ MG convenientemente
s s
2
x y x x y x x3 3x
2 + = + + ≥33 =33 = √
y z y y z yz xyz 3 xyz
x y 3x
⇒2 + ≥ √ ; similarmente
y z 3 xyz
y z 3y
2 + ≥ √
z x 3 xyz
z x 3z
2 + ≥ √
x y 3 xyz
x y z x+y+z
+ + ≥ √ , similarmente
y z x 3 xyz
x z y x+y+z
+ + ≥ √ .
z y x 3 xyz
1 1 1 1
+ 3 + 3 ≤ .
a3 b3 3 3
+ + 3abc b + c + 3abc c + a + 3abc abc
a2 + b2 ≥ 2ab, restando ab
⇔ a2 − ab + b2 ≥ ab, multiplicando por (a + b)
⇔ (a + b) (a2 − ab + b2 ) ≥ ab(a + b)
⇔ a3 + b3 ≥ ab(a + b), sumando abc
⇔ a3 + b3 + abc ≥ ab(a + b + c)
1 1
⇔ ≤
a3 3
+ b + abc ab(a + b + c)
abc abc
⇔ ≤
a3 3
+ b + abc ab(a + b + c)
abc c
⇔ ≤ , similarmente
a3 3
+ b + abc a+b+c
abc a
≤
b3 + c3 + abc a+b+c
abc b
3 3
≤
c + a + abc a+b+c
de donde
1 1 1 1
+ 3 + 3 ≤ .
a3 b3 3 3
+ + 3abc b + c + 3abc c + a + 3abc abc
2
181
a b c 3
+ + ≥ .
1 + b2 1 + c2 1 + a2 2
a b c a b c
2
+ 2
+ 2
≤ + +
1+b 1+c 1+a 2b 2c 2a
a b c 3
Pero aplicando MA ≥ MG, tenemos + + ≥ , y no podemos
2b 2c 2a 2
a b c 3
garantizar que + + ≥ .
1 + b2 1 + c2 1 + a2 2
Por ello utilizaremos otro procedimiento, veamos:
a ab2 ab2 ab
=a− ≥ a − =a− ,
1 + b2 1+b 2 2b 2
a ab
⇒ ≥ a − , similarmente
1 + b2 2
b bc
≥ b− ,
1 + c2 2
c ca
≥c− ,
1 + a2 2
1 3
3 − (ab + bc + ac) ≥ .
2 2
182 12. Soluciones
Sabemos que
a3 b3 c3 d3 a+b+c+d
2 2
+ 2 2
+ 2 2
+ 2 2
≥ .
a +b b +c c +d d +a 2
a3 ab2
= a − .
a2 + b2 a2 + b2
Pero
1 1
a2 + b2 ≥ 2ab ⇔ ≤
a2
+b 2 2ab
2
−ab −ab2
⇔ 2 ≥
a + b2 2ab
entonces
a3 ab2 b
2 2
=a− 2 2
≥a−
a +b a +b 2
183
a3 b
⇒ 2 2
≥ a − , similarmente
a +b 2
3
b c
2 2
≥b−
b +c 2
3
c d
2 2
≥c−
c +d 2
3
d a
2 2
≥ d−
d +a 2
a3 b3 c3 d3 a+b+c+d
2 2
+ 2 2
+ 2 2
+ 2 2
≥ . [9, pág 28]
a +b b +c c +d d +a 2
2
41. Sean a, b, c, d números reales positivos tales que abcd = 1, pruebe que
1 1 1
S = + +
1 + ab + bc + ca 1 + bc + cd + ab 1 + cd + da + ac
1
+ ≤ 1.
1 + da + ab + bd
1 1 1 1 1 1
+ + ≥ √ +√ +√
a b c ab ac bc
√ √ √
cd bd ad
= √ +√ +√
abcd abcd abcd
√ √ √
= cd + bd + ad
√ √ √ √
= d( c + b + a)
√ √ √ √
bc + ac + ab d( a + b + c)
⇒ ≥
abc · d d
√ √ √
a+ b+ c
⇒ bc + ac + ab ≥ √
d
184 12. Soluciones
√ √√ √
a+b+ c+ d
⇒ 1 + bc + ac + ab ≥ √
d
√
1 d
⇒ ≤ √ √ √ √ , similarmente
1 + bc + ac + ab a+ b+ c+ d
√
1 a
≤ √ √ √ √
1 + bc + cd + db a+ b+ c+ d
√
1 b
≤ √ √ √ √
1 + cd + da + ac a+ b+ c+ d
√
1 c
≤ √ √ √ √
1 + da + ab + bd a+ b+ c+ d
S ≤ 1.
1 1 1
+ + ≥ 1.
2ab2 +1 2bc2 2
+ 1 2ca + 1
como
a2 c + b2 a + bc2 p
≥ 3 (abc)3
3
⇔ a c + b a + bc2 ≥ 3abc
2 2
185
luego
basta demostrar
a+b+c √
≥ 3 abc
3
⇒ 1 ≥ abc; pues a + b + c = 3
⇒ abc ≤ 1.
(1 − abc)(2abc + 1)2 ≥ 0.
√ √ √ √ √ √
a x, b y, c z, yz, zx, xy
186 12. Soluciones
en efecto se tiene:
√ √ √
(a2 x + b2 y + c2 z) (yz + zx + xy) ≥ (a xyz + b xyz + c xyz)2
(a2 x + b2 y + c2 z) (yz + zx + xy) ≥ xyz(a + b + c)2
(a2 x + b2 y + c2 z) (yz + zx + xy) ≥ xyz(x + y + z)2
≥ xyz(3(xy + xz + yz))
⇒ a2 x + b2 y + c2 z ≥ 3xyz; similarmente
ax2 + by 2 + cz 2 ≥ 3abc.
Sumando
a2 (b + c − a) b2 (c + a − b) c2 (a + b − c) ab + bc + ca
+ + ≤ .
b+c c+a a+b 2
2
a4 b4 c4 (a2 + b2 + c2 )
+ + ≥
ab + ac bc + ba ac + bc 2(ab + ac + bc)
2
a4 b4 c4 (a2 + b2 + c2 )
⇔ − + + ≤−
ab + ac bc + ba ac + bc 2(ab + ac + bc)
187
En efecto equivale a:
2
2(ab + ac + bc) (a2 + b2 + c2 ) − (a2 + b2 + c2 ) ≤ (ab + ac + bc)2
2yx − x2 ≤ y 2
⇔ 0 ≤ x2 − 2xy + y 2
⇔ (x − y)2 ≥ 0.
√ p (1 + x) + (1 − x + x2 ) x2
1 + x3 = (1 + x) (1 − x + x2 ) ≤ =1+
2 2
√ x2
⇒ 1 + x3 ≤ 1 + .
2
188 12. Soluciones
Como
s v
a3 u 1 1
= u 3 = s
a3 + (b + c)3 ut b+c
b+c
3
1+ 1+
a a
1 1 a2
≥ 2 ≥ =
1 b+c b2 + c2 a2 + b2 + c2
1+ 1+
2 a a2
sumando tenemos:
s s s
3 3
a b c3
+ + ≥ 1.
a + (b + c)3
3 b + (c + a)3
3 c3 + (a + b)3
1
Prueba. Sumando a cada fracción tenemos la desigualdad equivalente
2
a la anterior
a−b 1 b−c 1 c−d 1
+ + + + +
a + 2b + c 2 b + 2c + d 2 c + 2d + a 2
d−a 1
+ + ≥2
d + 2a + b 2
efectuando
3a + c 3b + d 3c + a 3d + b
+ + + ≥4
a + 2b + c b + 2c + d c + 2d + a d + 2a + b
16(a + b + c + d)2
= = 4.
4(a + b + c + d)2
√ √ √
a2 − bc b + c + b2 − ca c + a + c2 − ab a + b ≥ 0.
m n p
√ +√ +√ ≥ 0, con m + n + p = 0.
b+c c+a a+b
m (m + p) p
√ − √ +√ ≥0
b+c c+a a+b
1 1 1 1
⇔ m √ −√ +p √ −√ ≥0
b+c c+a a+b c+a
1 1 1 1
m √
⇔ |{z} −√ + (−p) √ − √ ≥0
b+c c + a | {z } c+a b + a
(+) | {z } (+) | {z }
(+) (+)
b c a
Haciendo = x, = y, = z, tenemos:
a b c
1 1 1 3
2
+ 2
+ 2
≥ , con xyz = 1.
(1 + x) (1 + y) (1 + z) 4
191
Veamos que
1 1 1
2
+ 2
≥ ; x, y > 0.
(1 + x) (1 + y) 1 + xy
1 1 1 z z
+ ≥ = =
(1 + x)2 (1 + y)2 1 + xy z + xyz z+1
1 1 1 z 1
⇒ 2
+ 2
+ 2
≥ +
(1 + x) (1 + y) (1 + z) z + 1 (1 + z)2
z2 + z + 1
=
(1 + z)2
(z 2 + 2z + 1) − z
=
(1 + z)2
z
= 1− 2
z + 2z + 1
1
= 1−
1
z+ +2
z
1 1 1 3
∴ 2
+ 2
+ 2
≥ .
(1 + x) (1 + y) (1 + z) 4
2
192 12. Soluciones
1 1 1 1
+ + + = 1,
1+a 1+b 1+c 1+d
Prueba. Haciendo
1 1−x
= x ⇒ a =
1+a x
1 1−y
= y ⇒ b =
1+b y
1 1−z
= z ⇒ c =
1+c z
1 1−w
= w ⇒ d =
1+d w
entonces tenemos que
x+y+z+w =1
y demostraremos que
1−x 1−y 1−z 1−w
≥ 81
x y z w
⇔ (y + z + w)(x + z + w)(x + y + w)(x + y + z) ≥ 81xyzw
como
√
y + z + w ≥ 3 3 yzw
√
x + z + w ≥ 3 3 xzw
√
x + y + w ≥ 3 3 xyw
√
x + y + z ≥ 3 3 xyz
p
⇒ (y + z + w)(x + z + w)(x + y + w)(x + y + z) ≥ 81 3 (xyzw)3
(y + z + w)(x + z + w)(x + y + w)(x + y + z) ≥ 81xyzw.
2
193
a b c
+ + ≥ 1.
3a − b + c 3b − c + a 3c − a + b
Prueba.
a b c
S = + +
3a − b + c 3b − c + a 3c − a + b
4a 4b 4c
4S = + +
3a − b + c 3b − c + a 3c − a + b
a+b−c b+c−a c+a−b
4S = 1 + +1+ +1+
3a − b + c 3b − c + a 3c − a + b
(a + b − c)2 (b + c − a)2
4S = 3 + +
(a + b − c)(3a − b + c) (b + c − a)(3b − c + a)
(c + a − b)2
+
(c + a − b)(3c − a + b)
(a+b−c+b+c−a+c+a−b)2
4S ≥ 3 + (a+b−c)(3a−b+c)+(b+c−a)(3b−c+a)+(c+a−b)(3c−a+b)
(a + b + c)2 (a + b + c)2
= 3+ = 3 +
a2 + b2 + c2 + 2(ab + ac + bc) (a + b + c)2
= 3.
Luego
4S ≥ 4
∴ S ≥ 1.
2
194 12. Soluciones
Prueba. Como
(x + y + z + t)3 = x3 + y 3 + z 3 + t3 + 3x2 (y + z + t)
+3y 2 (x + z + t) + 3z 2 (x + y + t)
+3t2 (x + y + z) + 6(xyz + xyt + xzt + yzt)
luego
16 16 16 16
Haciendo x 21 = a, y 21 = b, z 21 = c, t 21 = d.
Tenemos
21 21 21 21 5 √
3
a 16 + b 16 + c 16 + d 16 ≥ a 16 · a3 + 63bcd
de donde
21
a a 16
√
3
≥ 21 21 21 21 .
3
a + 63bcd a 16 + b 16 + c 16 + d 16
2
195
a b c d
A= √
3
+√
3
+√
3
+√
3
≥ 1.
a3 + 63bcd b3 + 63acd c3 + 63bda d3 + 63abc
21
a a 16
√
3
≥ 21 21 21 21
a3 + 63bcd a 16 + b 16 + c 16 + d 16
21
b b 16
√
3 3
≥ 21 21 21 21
b + 63acd a 16 + b 16 + c 16 + d 16
21
c c 16
√
3 3
≥ 21 21 21 21
c + 63adb a 16 + b 16 + c 16 + d 16
21
d d 16
√
3
≥ 21 21 21 21
3
d + 63abc a 16 + b 16 + c 16 + d 16
sumando obtenemos
A ≥ 1.
Prueba. Sin pérdida de generalidad, sea a = máx {a, b, c}, entonces debe-
mos probar
r
3 a3 + b3 + c3 + 3abc
≥a
2
196 12. Soluciones
En efecto tenemos
a3 + b3 + c3 + 3abc ≥ 2a3
⇔ b3 + c3 − a3 + 3abc ≥ 0
⇔ b3 + c3 + (−a)3 + 3abc ≥ 0, (Identidad de Gauss)
⇔ (b + c − a)(b2 + c2 + a2 + bc − ab − ac) ≥ 0
1
⇔ (b + c − a)(2b2 + 2c2 + 2a2 + 2bc − 2ab − 2ac) ≥ 0
2
1
⇔ c − a})((a − b)2 + (a − c)2 + (b − c)2 ) ≥ 0
(b| + {z
2
(+)
pues a, b, c son los lados de un triángulo, por lo tanto está última desigualdad
es verdadera. 2
(a + b)2 (a + b)2
>
a2 + b2 + c2 + ab a2 + b2 + c2 + ab + ac + bc
2
(b + c) (b + c)2
>
a2 + b2 + c2 + bc a2 + b2 + c2 + ab + ac + bc
2
(c + a) (c + a)2
>
a2 + b2 + c2 + ca a2 + b2 + c2 + ab + ac + bc
2 (a2 + b2 + c2 + ab + ac + bc)
S> = 2.
a2 + b2 + c2 + ab + ac + bc
2
197
55. Dados a, b, c números reales tales que a, b, c ∈ [0; 1], pruebe que
a b c
+ + + (1 − a)(1 − b)(1 − c) ≤ 1.
b+c+1 c+a+1 a+b+1
(b + c + 1) + (1 − b) + (1 − c) p
≥ 3 (b + c + 1)(1 − b)(1 − c)
3
⇒ (b + c + 1)(1 − b)(1 − c) ≤ 1
1
⇒ (1 − b)(1 − c) ≤
b+c+1
1−a
⇒ (1 − a)(1 − b)(1 − c) ≤
b+c+1
a 1
⇒ + (1 − a)(1 − b)(1 − c) ≤ , y además
b+c+1 b+c+1
b b
≤
c+a+1 c+b+1
c c
≤
a+b+1 c+b+1
a b c
+ + + (1 − a)(1 − b)(1 − c) ≤ 1.
b+c+1 c+a+1 a+b+1
a 1
= ,
b+c+1 b+c+1
b b
≤ ,
c+2 c+b+1
c c
≤ ,
b+2 c+b+1
198 12. Soluciones
a b c
+ + ≤ 1.
b+c+1 c+a+1 a+b+1
En este caso la igualdad ocurre si y sólo si (a, b, c) = (1, 1, 1), (a, b, c) = (1, 1, 0),
(a, b, c) = (1, 0, 0), o sus respectivas permutaciones. 2
a b c 3
+ + ≤ .
2a + b + c 2b + c + a 2c + a + b 4
√ √ √ √
1 − ab + 1 − bc + 1 − ca ≥ 6. (12.10)
199
usando el dato a2 + b2 + c2 = 1;
1 2
1 − ab = [a + b2 + c2 + 1] − ab
2
1 2 1
= (c + 1) + (a2 + b2 − 2ab)
2 2
1 2 1 1
= (c + 1) + (a − b)2 ≥ (c2 + 1)
2 2 2
1 2
⇒ 1 − ab ≥ (c + 1) ; similarmente
2
1 2
1 − bc ≥ (a + 1)
2
1 2
1 − ca ≥ (b + 1)
2
1 2 1
(1 − ab)(1 − bc) ≥ (a + 1) (c2 + 1) ≥ (ac + 1)2
4 4
p 1
⇒ (1 − ab)(1 − bc) ≥ (ac + 1); similarmente
2
p 1
(1 − ab)(1 − ac) ≥ (bc + 1)
2
p 1
(1 − bc)(1 − ac) ≥ (ab + 1)
2
hp p p i
⇒ 2 (1 − ab)(1 − bc) + (1 − ab)(1 − ca) + (1 − bc)(1 − ca)
≥ ab + bc + ca + 3
200 12. Soluciones
⇔ a4 − a3 (b + c) + a2 bc − b3 (c + a) + b2 ca − c3 (a + b) + c2 ab ≥ 0
(ab + bc + ca)2
Prueba. Aplicando MA ≥ MG a los números (a+b+c), ,
a+b+c
tenemos
(ab + bc + ca)2 s
(a + b + c) + (ab + bc + ca)2
a+b+c ≥ (a + b + c)
2 a+b+c
4(ab + bc + ca)2
⇔ 4(a + b + c) + ≥ 8(ab + bc + ca).
a+b+c
4(ab + bc + ca)2
a3 + b3 + c3 + 4(a + b + c) + 9abc ≥ 4(a + b + c) +
a+b+c
4(ab + bc + ca)2
⇔ a3 + b3 + c3 + 9abc ≥
a+b+c
⇔ a + ab + ac + ba + b + bc3 + ca3 + cb3 + c4 + 9abc(a + b + c)
4 3 3 3 4
2
202 12. Soluciones
Haciendo a + b + c = x.
Entonces
p √
M≥ x2 + (3 − x)2 = 2x2 − 6x + 9
s
2
9 9
= 2 x − 3x + +
4 2
s 2
3 9
= 2 x− +
2 2
r √
9 3 2
≥ =
2 2
√
3 2
∴ M≥ .
2
2
Prueba. Efectuando
(m + n + p + q)2
= m2 + n2 + p2 + q 2 + 2(mn + mp + mq + np + nq + pq)
= m2 + n2 + p2 + q 2 + mn + mn + mp + mp + · · · + pq + pq)
| {z }
p
≥ m2 + 15 15 m6 n8 p8 q 8
= m2 + 15m6/15 n8/15 p8/15 q 8/15
⇒ m+n+p+q
p
≥ m2 + 15m6/15 · n8/15 · p8/15 · q 8/15
r 24
6
= m 15 m + 15n
15 8/15 ·p 8/15 ·q 8/15
r
1 8 3
= m5 m 15 + 15n8/15 · p8/15 · q 8/15
⇒ m+n+p+q
r 3
1 8
≥ m5 m 15 + 15n8/15 · p8/15 · q 8/15
Haciendo
m8/15 = a; p8/15 = c
n8/15 = b; q 8/15 = d.
Tenemos:
√
a15/8 + b15/8 + c15/8 + d15/8 ≥ a3/8 a3 + 15bcd
a3/2 a15/8
⇒ √ ≥ 15/8
a3 + 15bcd a + b15/8 + c15/8 + d15/8
204 12. Soluciones
r
a3 a15/8
⇒ ≥ ; similarmente
a3 + 15bcd a15/8 + b15/8 + c15/8 + d15/8
r
b3 b15/8
≥
b3 + 15acd a15/8 + b15/8 + c15/8 + d15/8
r
c3 c15/8
≥
c3 + 15abd a15/8 + b15/8 + c15/8 + d15/8
r
d3 d15/8
≥
d3 + 15abc a15/8 + b15/8 + c15/8 + d15/8
1 1 1
+ 2 + 2 ≤ 1.
a2 +2 b +2 c +2
Como:
2 2 2−x 2 2 2 2−x
x + (3 − x) − 4yz − 4 ≥ x + (3 − x) − (y + z) −4
4 4
2 2 2 2−x
= x + (3 − x) − (3 − x) −4
4
a3 b3 c3 3 2 1
+ + ≥ a + b2 + c2 − (ab + bc + ca).
a+b b+c c+a 4 4
a3 a(a + b) r
+ a3 a(a + b)
a+b 4 ≥ ·
2 a+b 4
a3 a2 + ab
+ ≥ a2
a+b 4
a3 3 2 1
≥ a − ab; similarmente
a+b 4 4
b3 3 2 1
≥ b − bc
b+c 4 4
c3 3 2 1
≥ c − ca
c+a 4 4
206 12. Soluciones
sumando tenemos:
a3 b3 c3 3 2 1
+ + ≥ a + b2 + c2 − (ab + bc + ca).
a+b b+c c+a 4 4
2
ab bc ca 1
S= + + ≤ (a + b + c).
2c + a + b 2a + b + c 2b + c + a 4
4mn ≤ (m + n)2
4 m+n
⇔ ≤
m+n mn
4 1 1
⇔ ≤ +
m+n m n
1
∴ S ≤ (a + b + c).
4
2
207
65. Si a ≥ b ≥ c ≥ d ≥ e, entonces
Prueba. Veamos
pues c ≥ d ∧ b ≥ c.
La igualdad ocurre si
a+d+e a+b+e
b=c= ∨ c=d= .
2 2
2
1 1 1
1≥ + +
z x y
208 12. Soluciones
1 1 1
Haciendo = a, = b, = c
x y z
⇒ a + b + c ≤ 1,
como
1 ≥ a + b + c ⇒ 1 ≥ (a + b + c)2
⇔ 1 ≥ a2 + b2 + c2 + 2(ab + bc + ca) ≥ 3(ab + bc + ca)
⇔ 1 ≥ 3(ab + bc + ca).
2
x y 2
√ +p ≥√ .
1 − x2 1 − y2 3
x y x2 y2 (x + y)2
√ +p = √ + p ≥ √ p
1 − x2 1 − y2 x 1 − x2 y 1 − y 2 x 1 − x2 + y 1 − y 2
1
= √ p
x 1 − x2 + y 1 − y 2
√
1 2 √ p 3
√ p ≥ √ ⇔ x 1−x +y 1−y ≤
2 2
x 1 − x2 + y 1 − y 2 3 2
√
√ √ √ p 3
⇔ x x − x3 + y y − y 3 ≤
2
209
aplicando Cauchy-Schwarz
√ √ √ p 2 √ √ 2 √ 2 p 2
3 3 2 3 3
x x−x + y y−y ≤ x + y x−x + y−y
= (x + y) (x + y − (x3 + y 3 ))
= 1 − (x3 + y 3 )
de donde
2ab a+b
≤
a+b 2
2bc b+c
≤
b+c 2
2ca c+a
≤ ,
c+a 2
√
x4 + y 4 + z 2 ≥ 8xyz.
211
z2 z2
Prueba. Aplicando MA ≥ MG a los números x4 , y 4, ,
2 2
r
4 z2 z2
4 4
4 4
z2 z2
x +y + + ≥ 4 x ·y · ·
2 2 2 2
xyz 4
⇔ x4 + y 4 + z 2 ≥ 4 · √4
= √ xyz
4 2
√
∴ x4 + y 4 + z 2 ≥ 8xyz.
como
r
1 1 1 1 3
+ + ≥ 33 = √
a b c abc 3
abc
r
1 1 1 1 3
+ + ≥ 33 2 2 2 = √ 2
ab ac bc abc 3
abc
3
1 1
= √
abc 3
abc
luego
3 3
1 1 1 3 3 1 1
1+ 1+ 1+ ≥ 1+ √ +√ 2+ √ = 1+ √
a b c 3
abc 3 abc 3
abc 3
abc
212 12. Soluciones
1 √3 1
≥ abc ⇔ √ ≥3
3 3
abc
1
⇔ 1+ √ 3
≥4
abc
3
1
⇔ 1+ √3
≥ 64
abc
de donde
1 1 1
+1 +1 +1 ≥ 64.
a b c
2
pruebe que
xy + yz + zx ≥ a2 + b2 + c2 .
ac bc
⇒ xy ≥ ab + c2 − − ; similarmente
2 2
ab bc
⇒ xz ≥ ac + b2 − −
2 2
ab ac
⇒ yz ≥ bc + a2 − −
2 2
sumando miembro a miembro, se tiene
xy + yz + zx ≥ a2 + b2 + c2 .
1 1 1
≥ ≥
1 + x + x2 + x3 1 + y + y2 + y3 1 + z + z2 + z3
= 0.
214 12. Soluciones
n X
X n
i · j · cos (ai − aj ) ≥ 0.
i=1 j=1
Prueba.
n X
X n
i · j (cos ai cos aj + sen ai sen aj )
i=1 j=1
n X
X n
= (i · j cos ai cos aj + i · j sen ai sen aj )
i=1 j=1
X n
n X n X
X n
= ij cos ai cos aj + ij sen ai sen aj
i=1 j=1 i=1 j=1
n
X n
X n
X n
X
= i cos ai j cos aj + i sen ai j sen aj
i=1 j=1 i=1 j=1
| {z } | {z }
n
X Xn n
X Xn
= i cos ai i cos ai + i sen ai i sen ai
i=1 i=1 i=1 i=1
n
!2 n
!2
X X
= i cos ai + i sen ai ≥ 0.
i=1 i=1
2
pruebe que
xy + yz + zx ≥ (a + b + c)2 .
215
ac bc
xy ≥ ab + c2 + +
2 2
ab bc
xz ≥ ac + b2 + +
2 2
ab ac
yz ≥ bc + a2 + +
2 2
xy + yz + zx ≥ a2 + b2 + c2 + 2(ab + ac + bc)
∴ xy + yz + zx ≥ (a + b + c)2 .
a b c d e f
+ + + + + ≥ 3.
b+c c+d d+e e+f f +a a+b
a b c d e f
S = + + + + +
b+c c+d d+e e+f f +a a+b
a2 b2 c2 d2 e2 f2
= + + + + +
ab + ac bc + bd cd + ce ed + f d ef + ea af + bf
(a + b + c + d + e + f )2
≥
ab + ac + bc + bd + cd + ce + ed + f d + ef + ea + af + bf
(a + b + c + d + e + f )2
=
M
216 12. Soluciones
donde
M = ab + ac + bc + bd + cd + ce + ed + f d + ef + ea + af + bf
2M = (a + b + c + d + e + f )2 − (a + d)2 − (b + e)2 − (c + f )2 (12.12)
(a + d)2 + (b + e)2 + (c + f )2 12 + 12 + 12 ≥ (a + b + c + d + e + f )2
3 (a + d)2 + (b + e)2 + (c + f )2 ≥ (a + b + c + d + e + f )2 (12.13)
De (12.12) tenemos
(a + d)2 + (b + e)2 + (c + f )2 = (a + b + c + d + e + f )2 − 2M
3 [(a + d)2 + (b + e)2 + (c + f )2 ] = 3(a + b + c + d + e + f )2 − 6M
3(a + b + c + d + e + f )2 − 6M ≥ (a + b + c + d + e + f )2
2(a + b + c + d + e + f )2 ≥ 6M
(a + b + c + d + e + f )2
≥ 3.
M
a b c d e f
∴ + + + + + ≥ 3.
b+c c+d d+e e+f f +a a+b
2
a3 + b3 + c3 x3 + y 3 + z 3 t3 + u3 + v 3 ≥ (axt + byu + czv)3 .
217
a3 b3 c3
+ + +
a3 + b3 + c3 a3 + b3 + c3 a3 + b3 + c3
x3 y3 z3
+ + +
x3 + y 3 + z 3 x3 + y 3 + z 3 x3 + y 3 + z 3
t3 u3 v3
+ + =3
t3 + u3 + v 3 t3 + u3 + v 3 t3 + u3 + v 3
a3 x3 t3
+ +
a3 + b3 + c3 x3 + y 3 + z 3 t3 + u3 + v 3
3axt
≥ p ,
3
(a3 + b3 + c3 ) (x3 + y 3 + z 3 ) (t3 + u3 + v 3 )
b3 y3 u3
+ +
a3 + b3 + c3 x3 + y 3 + z 3 t3 + u3 + v 3
3byu
≥ p ,
3
(a3 + b3 + c3 ) (x3 + y 3 + z 3 ) (t3 + u3 + v 3 )
c3 z3 v3
+ +
a3 + b3 + c3 x3 + y 3 + z 3 t3 + u3 + v 3
3czv
≥ p ,
3
(a + b + c ) (x + y 3 + z 3 ) (t3 + u3 + v 3 )
3 3 3 3
a b c
√ +√ +√ ≥ 1.
a2 + 8bc b2 + 8ca c2 + 8ab
a b c
∴ √ +√ +√ ≥ 1.
a2 + 8bc b2 + 8ca c2 + 8ab
2
1 1 1
79. (APMO 2002). Sean a, b y c números reales positivos que satisfacen + + =
a b c
1, pruebe que
√ √ √ √ √ √ √
a + bc + b + ca + c + ab ≥ abc + a+ b+ c.
219
Prueba. Como
1 1 1
+ + =1
a b c
√ 1 1 1 √
⇔ abc + + = abc
a b c
r r r
bc ac ab √
⇔ + + = abc.
a b c
rr r
√ √ √ bc ac ab √ √ √
a + bc + b + ca + c + ab ≥ + + + a+ b+ c
a b c
r ! r
√ √ √ bc √ ac √
a + bc + b + ca + c + ab ≥ + a + + b
a b
r !
ab √
+ + c
c
2
220 12. Soluciones
80. Sean a, b, c números reales positivos que satisfacen abc = 1, pruebe que
3 6
1+ ≥ .
a+b+c ab + ac + bc
1 1 1
Prueba. Haciendo = x, = y, = z ⇒ xyz = 1.
a b c
La desigualdad es equivalente a:
3 6
1+ ≥
1 1 1 1 1 1
+ + + +
x y z xy xz yz
3 6
⇔ 1+ ≥ (12.14)
xy + xz + yz x+y+z
Pero
9 6
1+ 2
≥
(x + y + z) x+y+z
2
3
⇔ 1− ≥ 0.
x+y+z
2
1 1 1 1−c c 1−a a 1−b
(abc + xyz) + + = + + + +
ay bz cx a 1−b b 1−c c
b
+ −3
1−a
1−c a c 1−b
= + + +
a 1−c 1−b c
1−a b
+ + −3
b 1−a
≥ 2 + 2 + 2 − 3 = 3.
2
x+y+z xy + yz + zx
√ ≥p p √ .
3 3 x2 + xy + y 2 + y 2 + yz + z 2 + z 2 + zx + x2
222 12. Soluciones
Prueba. Como
p p √
x2 + xy + y 2 + y 2 + yz + z 2 + z 2 + zx + x2
√
3
≥ (x + y + y + z + z + x)
√2
= 3(x + y + z)
1
⇔ p p √
x + xy + y + y + yz + z 2 + z 2 + zx + x2
2 2 2
1
≤ √
3(x + y + z)
xy + yz + zx
⇔ p p √
x + xy + y + y 2 + yz + z 2 + z 2 + zx + x2
2 2
xy + yz + zx
≤ √ (12.16)
3(x + y + z)
Pero
x2 + y 2 + z 2 ≥ xy + xz + yz
⇔ (x + y + z)2 ≥ 3(xy + xz + yz)
x+y+z xy + xz + yz
√ ≥ √ (12.17)
3 3 3(x + y + z)
223
xy + yz + zx x+y+z
p p √ ≤ √ .
x2 + xy + y2 2 2 2
+ y + yz + z + z + zx + x2 3 3
2
√ p
≤ 2 2(1 + 1)
√
= 2 2
p p √
∴ (x + z)2 + (y + t)2 + (x − z)2 + (y − t)2 ≤ 2 2.
2
84. Dados a, b, c números reales tales que a2 +b2 +c2 ≤ 8, pruebe que ab+bc+2ca ≥
−8.
Prueba. Como
8 ≥ a2 + b2 + c2
⇔ 8 + ab + bc + 2ca ≥ a2 + b2 + c2 + ab + bc + 2ca
b2 3b2
= a2 + + c2 + ab + bc + 2ca +
| 4 {z } 4
2
b 3b2
= a+ +c + ≥0
2 4
224 12. Soluciones
b
a+ + c = 0 ∧ b = 0 ∧ a2 + b2 + c2 = 8,
2
de donde
b4 c ≤ a3 bc y c4 a ≤ c2 a3 ≤ ca4
3c c
y desde que ≥ , tenemos:
4 2
a4 b + b4 c + c4 a ≤ a4 b + a3 bc + c4 a
4 3 c4 a c4 a
= a b + a bc + +
2 2
4 2 3
ca c a
≤ a4 b + a3 bc + +
2 2
3
ac
= a3 b(a + c) + (a + c)
2
3 c 3 3c
= a (a + c) b + ≤ a (a + c) b +
2 4
225
3c
⇒ a4 b + b4 c + c4 a ≤ a3 (a + c) b +
4
4 a a a a+c 3c
= 4 · · · · b+
4 4 4 4 4
a a a a+c 3c 5
+ + + +b+
4
≤ 44 4 4 4 4
5
5 5
4 a+b+c 41 256
= 4 =4 = .
5 5 3125
2
a b c
+ + ≥ 3.
b+c−a c+a−b a+b−c
m n
es verdadera pues + ≥ 2, ∀ m, n ∈ R+ . 2
n m
87. Sea n un número entero ≥ 2, pruebe que
!4
X X
8 xi xj x2i + x2j ≤ xi .
1≤i<j≤n 1≤i≤n
226 12. Soluciones
Prueba. Recordar
!4 !2 2 !2
X X X X
xi = xi = x2i + 2 xi xj
1≤i≤n 1≤i≤n 1≤i≤n 1≤i<j≤n
! !
X X
≥ 4 x2i 2 xi xj
1≤i≤n 1≤i<j≤n
!
X X
= 8 x2i xi xj
1≤i≤n 1≤i<j≤n
X
= 8 x21 + x22 + · · · + x2n xi xj
1≤i<j≤n
X
= 8 xi xj x21 + x22 + · · · + x2n
1≤i<j≤n
X
≥ 8 xi xj x2i + x2j .
1≤i<j≤n
S S S n2
+ +···+ ≥ .
S − a1 S − a2 S − an n−1
227
(S − a1 ) + (S − a2 ) + · · · + (S − an ) n
≥
n 1 1 1
+ +···+
S − a1 S − a2 S − an
1 1 1
⇔ nS − a1 + a2 + · · · + an + +···+ ≥ n2
| {z } S − a1 S − a2 S − an
S
1 1 1
⇔ (n − 1)S + +···+ ≥ n2
S − a1 S − a2 S − an
S S S n2
⇔ + +···+ ≥ .
S − a1 S − a2 S − an n−1
√ √ √
a 1 + b3 + b 1 + c3 + c 1 + a3 ≤ 5.
√ p
a 1 + b3 = a (1 + b) (1 − b + b2 )
La desigualdad es equivalente a:
p p p
a (1 + b) (1 − b + b2 ) + b (1 + c) (1 − c + c2 ) + c (1 + a) (1 − a + a2 ) ≤ 5
pero
p (1 + b) + (1 − b + b2 )
a (1 + b) (1 − b + b2 ) ≤ a
2
p 1
⇔ a (1 + b) (1 − b + b2 ) ≤ (a) (2 + b2 )
2
228 12. Soluciones
1 1 1
a (2 + b2 ) + b (2 + c2 ) + c (2 + a2 ) ≤ 5
2 2 2
⇔ 2(a b + }c) + ab + bc2 + ca2 ≤ 10
| +{z
2
c≤b≤a
⇒ a(b − a)(b − c) ≤ 0
⇔ ab2 + a2 c ≤ abc + a2 b
abc + a2 b + bc2 ≤ 4
⇔ b (ac + a2 + c2 ) ≤ 4
Pero
2
a+c a+c 2(a + c)
1+ 1+ ≥ 1+
a+b c+d a+b+c+d
2
b+d d+b 2(b + d)
1+ 1+ ≥ 1+
b+c d+a a+b+c+d
a+c b+d c+a d+b
⇒ 1+ 1+ 1+ 1+
a+b b+c c+d d+a
2 2
2(a + c) 2(b + d)
≥ 1+ 1+
a+b+c+d a+b+c+d
2 2
2(a + c) 2(b + d)
1+ 1+ ≥9
a+b+c+d a+b+c+d
2(a + c) 2(b + d)
⇔ 1+ 1+ ≥ 3.
a+b+c+d a+b+c+d
230 12. Soluciones
2(m + n) 4mn
⇔ 1+ + ≥3
m+n (m + n)2
4mn
⇔ 3+ ≥3
(m + n)2
4mn
⇔ ≥ 0.
(m + n)2
b c a
Haciendo = x2 , = y 2 , = z 2 , con x, y, z positivos ⇒ (xyz)2 = 1
a b c
Reemplazando tenemos:
r r r
2 2 2
+ + ≤3
1 + x2 1 + y2 1 + z2
231
Pero
r r 2
2 2 2 2 2 2
·1+ ·1 ≤ (1 + 1 ) +
1 + x2 1 + y2 1 + x2 1 + y 2
1 1
= 4 +
1 + x2 1 + y 2
1 − (xy)2
= 4 1+
(1 + x2 ) (1 + y 2 )
1 − (xy)2
≤ 4 1+
(1 + xy)2
8 8z
= =
1 + xy z+1
r r √
2 2 2 2z
⇒ + ≤ √
1 + x2 1 + y2 z+1
√ r
2 2z 2
√ + ≤3
z+1 1 + z2
Pero
(z + 1)2 ≤ 2 (1 + z 2 )
2 4
⇔ 2
≤
1+z (z + 1)2
r
2 2
⇔ 2
≤ ,
1+z (z + 1)
√
2 2z 2
√ + ≤3
z+1 z+1
232 12. Soluciones
p
⇔ 2 2z(z + 1) + 2 ≤ 3 + 3z
p p
⇔ 0 ≤ (3z + 1) − 2 2z(z + 1) = 2z − 2 2z(z + 1) + (z + 1)
√ √
⇔ 0 ≤ ( 2z − z + 1)2 . [8, pág 34]
2
1 1 1
+ +···+ = 1,
1 + x1 1 + x2 1 + xn
pruebe que
x1 x2 · · · xn ≥ (n − 1)n .
Prueba. De la condición
1 1 1 1
+ +···+ + =1
1 + x1 1 + x2 1 + xn−1 1 + xn
tenemos
1 1 1 1 xn
+ +···+ =1− =
1 + x1 1 + x2 1 + xn−1 1 + xn 1 + xn
1 1 1
+ +···+ r
1 + x1 1 + x2 1 + xn−1 1 1 1
≥ n−1
· ···
n−1 1 + x1 1 + x2 1 + xn−1
xn n−1
⇒ ≥ p
1 + xn (1 + x1 ) (1 + x2 ) · · · (1 + xn−1 )
n−1
xi n−1
⇒ ≥ v ; i = 1, 2, . . . , n
1 + xi u Y n
u
u (1 + xj )
u
u
n−1
t j=1
j 6= i
233
x1 x2 · · · xn ≥ (n − 1)n .
ab bc ca
+ + ≥ 3.
c a b
Aplicando MA ≥ MG:
r
a2 b2 b2 c2 a2 b2 b2 c2
+ 2 ≥ 2 · 2 = 2b2
c2 a c2 a
r
a2 b2 c2 a2 a2 b2 c2 a2
+ 2 ≥ 2 · 2 = 2a2
c2 b c 2 b
r
b2 c2 c2 a2 b2 c2 c2 a2
+ 2 ≥ 2 · 2 = 2c2
a2 b a2 b
Sumando:
a2 b2 b2 c2 c2 a2
+ 2 + 2 ≥ a2 + b2 + c2 .
c2 a b
2
234 12. Soluciones
a+b+c
aa · bb · cc ≥ (abc) 3 .
de donde
a+b+c
a b c 3
a · b · c ≥ (abc) .
a b c
Prueba. Haciendo = x, = y, = z, tenemos xyz = 1 y además
b c a
a+c 1 + xy 1−x
= =x+
b+c 1+y 1+y
pero
1 1 4 1 1 4
+ ≥ ; + ≥
b+c a+d (b + c) + (d + a) c + d a + b (c + d) + (a + b)
236 12. Soluciones
entonces
1 1 4(a + c)
(a + c) + ≥
b+c a+d a+b+c+d
1 1 4(b + d)
(b + d) + ≥
c+d a+b a+b+c+d
a3 b3 c3 3(ab + ac + bc)
2 2
+ 2 2
+ 2 2
≥ .
b − bc + b c − ac + a a − ab + b a+b+c
3(ab + ac + bc)
Prueba. Como (a+b+c)2 ≥ 3(ab+ac+bc) ⇒ a+b+c ≥ .
a+b+c
Demostraremos la siguiente desigualdad que es más fuerte
a3 b3 c3
+ + ≥ a + b + c.
b2 − bc + b2 c2 − ac + a2 a2 − ab + b2
a3 b3 c3
+ +
b2 − bc + b2 c2 − ac + a2 a2 − ab + b2
a4 b4 c4
= + +
ab2 − abc + ab2 bc2 − abc + a2 b a2 c − abc + b2 c
2
(a2 + b2 + c2 )
≥
ab2 + ac2 + bc2 + ba2 + a2 c + b2 c − 3abc
237
2
(a2 + b2 + c2 )
a4 + b4 + c4 + 2 (a2 b2 + a2 c2 + b2 c2 )
a4 + b4 + c4 + a2 bc + ab2 c + abc2
Haciendo
b c
= x, = y,
a b
d a
= z, = w.
c d
2 2 2 2
1 1 1 1
+ + + ≥1
1+x 1+y 1+z 1+w
con xyzw = 1.
Aplicando el problema 21, tenemos que
1 1 1
2 + 2 ≥ ; similarmente
(1 + x) (1 + y) 1 + xy
1 1 1 xy xy
2 + 2 ≥ = =
(1 + z) (1 + w) 1 + zw xy + xyzw xy + 1
1 1 1 1 1 + xy
2 + 2 + 2 + 2 ≥ =1
(1 + x) (1 + y) (1 + z) (1 + w) xy + 1
1 1 1 1
∴ 2 + 2 + 2 + ≥ 1.
(1 + x) (1 + y) (1 + z) (1 + w)2
3 1 − a + a2 1 − b + b2 1 − c + c2 ≥ 1 + abc + a2 b2 c2 .
239
2 (1 − a + a2 ) (1 − b + b2 )
+4ab
⇒ 2 (1 − a + a2 ) (1 − b + b2 ) ≥ 1 + a2 b2
1
⇔ (1 − a + a2 ) (1 − b + b2 ) ≥ (1 + a2 b2 )
2
3
⇔ 3 (1 − a + a2 ) (1 − b + b2 ) (1 − c + c2 ) ≥ (1 + a2 b2 ) (1 − c + c2 )
2
3
1 + a2 b2 1 − c + c2 ≥ 1 + abc + a2 b2 c2
2
3 (1 + a2 b2 ) c2 + 3 (1 + a2 b2 ) (1 − c) ≥ 2 + 2abc + 2a2 b2 c2
calculando su discriminante
2
∆ = (3a2 b2 + 2ab + 3) − 4 (3 + a2 b2 ) (1 + 3a2 b2 )
= −3[ab − 1]4 ≤ 0
(1 − a)(1 − b) ≥ 0
(1 − b)(1 − c) ≥ 0
(1 − c)(1 − a) ≥ 0.
101. Sean a, b, c números reales positivos tales que abc ≥ 1, pruebe que
a3 + b3 + c3 ≥ ab + ac + bc.
∴ a3 + b3 + c3 ≥ ab + ac + bc.
√ √ √ √ √ √
ab + c + bc + a + ca + b ≥ 1 + ab + bc + ca.
242 12. Soluciones
Prueba. Como
√
a + b ≥ 2 ab
√
⇔ a + b + c ≥ 2 ab + c
√
⇔ 1 ≥ 2 ab + c
√
⇔ c ≥ 2 abc + c2
√
⇔ ab + c ≥ ab + 2 abc + c2
√
⇔ ab + c ≥ ( ab + c)2
√ √
⇔ ab + c ≥ ab + c; similarmente
√ √
bc + a ≥ bc + a
√ √
ca + b ≥ ca + b
√ √ √ √ √ √
∴ ab + c + bc + a + ca + b ≥ 1 + ab + bc + ca.
103. Sean a, b números reales positivos tales que a + b ≤ 1, halle el mı́nimo valor
de f (a, b) = ab + (ab)−1 .
1
Solución. Como a, b > 0 entonces ab + ≥ 2 y decimos que el mı́nimo de
ab
f es 2, pero veamos cuándo ocurre este mı́nimo.
1
El mı́nimo ocurre si ab = =1
ab
entonces
√ a+b 1
1= ab ≤ ≤ , ¡contradicción!
2 2
Haciendo
1 1 1 1
h= ⇒h= ≥ 2 ≥ 2 = 4
ab ab a+b 1
2 2
h 4
= 4 1
α α ⇒ = ⇒ α = 16
1 1 α 4
=
h 4
Luego
r
1 h 1 15h h 1 15h
h+ = + + ≥ 2 · +
h 16 h 16 16 h 16
2 15h 2 15 × 4 17
= + ≥ + =
4 16 4 16 4
de donde
1 17
h+ ≥
h 4
entonces
1 17
ab + ≥
ab 4
17 1
∴ el mı́nimo es y ocurre cuando a = b = c = .
4 2
2
244 12. Soluciones
3
104. Sean a, b, c números reales positivos tales que a + b + c ≤ , halle el mı́nimo
r r r 2
1 1 1
de f (a, b, c) = a2 + 2 + b2 + 2 + c2 + 2 .
b c a
Solución. Aplicando MA ≥ MG:
sr r r
3 1 1 1
f (a, b, c) ≥ 3 a2 + 2 · b2 + 2 · c2 +
b c a2
s
1 1 1
= 3 6
a2 + 2 b2 + 2 c2 + 2
b c a
s r r r
6 1 1 1
≥ 3 2 a2 · 2 · 2 b2 · 2 · 2 c2 · 2 ·
b c a
√
= 3 2
√ √
entonces f (a, b, c) ≥ 3 2, el mı́nimo de f es 3 2 y ocurre cuando
1 1 1
a=b=c= = = = 1,
a b c
3
entonces a + b + c = 3, pero a + b + c ≤ ¡contradicción!
2
√
Esto significa que el mı́nimo no es 3 2.
Analizamos la expresión y aprovechando la forma simétrica estimamos que el
1
mı́nimo de f (a, b, c) ocurre cuando a = b = c = , luego como en el problema
2
anterior hacemos
1
a2 = b2 = c2 =
1 4
a=b=c= ⇒
2
1 = 1 = 1 = 4
αa2 αb2 αc2 α
1 4
⇒ = ⇒ α = 16
4 α
245
v v
u 1 1 u
u
f (a, b, c) = ua2 + + · · · + + ub2 + 1 + · · · + 1
2 2 u 2 2
t |16b {z 16b } t |16c {z 16c }
16 sumandos 16 sumandos
v
u 1 1
+u
uc2 + 2
+···+ 2
t |16a {z 16a }
16 sumandos
s r s r s r
2 2
17 a 17 b 17 c2
≥ 17 + 17 + 17
1616 b32 1616 c32 1616 a32
"r r r #
√ a b c
= 17 17 8 16
+ 17 8 16
+ 17 +
16 b 16 c 168 a16
sr
r
√ 3 17 abc √ 1
≥ 3 17 24 16 16 16
= 3 17 · 17
16 a b c 16 (abc)5
8
√ √ √
3 17 3 17 3 17
= 17
p = 17p = 17p
168 (abc)5 217 · 215 · (abc)5 2 ((2a)(2b)(2c))5
√ √
3 17 3 17
≥ r ≥ ;
17 2a + 2b + 2c 5 2
2 ( )
3
1
como vemos para a = b = c = ocurre el mı́nimo de f (a, b, c) y dicho mı́nimo
√ 2
3 17
es . 2
2
105. Sean a, b, c números reales tales que a ≥ 2, b ≥ 6 y c ≥ 12, halle el máximo
valor de
√ √ √
bc a − 2 + ca 3 b − 6 + ab 4 c − 12
f (a, b, c) = .
abc
246 12. Soluciones
Solución.
√ bc p bc (a − 2) + 2 abc
bc a − 2 = √ (a − 2) · 2 ≤ √ · = √
2 2 2 2 2
√ ca 3p ca (b − 6) + 3 + 3 abc
ca 3 b − 6 = √ (b − 6) · 9 ≤ √ · = √
3
9 3
9 3 339
√ ab p ab (c − 12) + 4 + 4 + 4 abc
ab 4 c − 12 = √ 4
(c − 12) · 64 ≤ √ · = √
4
64 4
64 4 4 4 64
abc
= √
8 2
entonces
1 abc abc abc 5 1
f (a, b, c) ≤ √ + √ + √ = √ + √
abc 2 2 339 8 2 8 2 339
a − 2 = 2, b − 6 = 3, c − 12 = 4
a = 4, b = 9, c = 16
5 1
∴ máx f (a, b, c) = √ + √ .
8 2 339
2
a3 b3 c3
106. Sean a, b, c números reales positivos, pruebe que + + ≥ ab + bc + ca.
b c a
Prueba. Aplicando MA ≥ MG:
r
a3 b3 3 a
3
b3
+ + bc ≥ 3 · · bc = 3ab
b c b c
r
b3 c3 3 b
3
c3
+ + ca ≥ 3 · · ca = 3bc
c a c a
r
3 3 3
c a 3 c a3
+ + ab ≥ 3 · · ab = 3ca
a b a b
247
a3 b3 c3
de donde + + ≥ ab + bc + ca.
b c a
La igualdad ocurre si y sólo si a = b = c. 2
a b c d
f (a, b, c, d) = + + +
b+c+d c+d+a d+a+b a+b+c
b+c+d c+d+a d+a+b a+b+c
+ + + + .
a b c d
q
a b c d b+c+d c+d+a d+a+b a+b+c
f (a, b, c, d) ≥ 8 8 b+c+d · c+d+a
· d+a+b
· a+b+c
· a
· b
· c
· d
f (a, b, c, d) ≥ 8
⇒ mı́n f = 8.
¿Cuándo ocurre tal mı́nimo?
a= b+c+d
b = c + d + a
mı́n f = 8 ⇔
c = d+a+b
d = a + b + c
sumando tenemos
|a + b {z
+ c + d} = 3(a
| + b {z
+ c + d})
(+) (+)
⇒ 1 = 3 imposible.
248 12. Soluciones
Luego el mı́nimo de f no es 8.
Sean a = b = c = d > 0
a b c d 1
⇒ = = = =
b+c+d c+d+a d+a+b a+b+c 3
b+c+d c+d+a d+a+b a+b+c 3
= = = =
αa αb αc αd α
1 3
igualando = ⇒ α = 9.
3 α
Esto significa que podemos agrupar de esta manera:
a b c d b+c+d c+d+a
+ + + + +
b+c+d c+d+a d+a+b a+b+c 9a 9b
d+a+b a+b+c
+ +
9c 9d
8 b+c+d c+d+a d+a+b a+b+c
+ + + +
9 a b c d
q
a b c d
≥ 8 8 b+c+d · c+d+a · d+a+b · a+b+c · b+c+d
9a
· c+d+a
9b
· d+a+b
9c
· a+b+c
9d
8 b c d c d a d a b a b c
+ + + + + + + + + + + +
9 a a a b b b c c c d d d
r
8 8 b c d c d a d a b a b c
≥ + · 12 12 · · · · · · · · · · ·
3 9 a a a b b b c c c d d d
8 32 40
= + =
3 3 3
40
∴ fmı́n = y ocurre cuando a = b = c = d > 0.
3
2
a3 (b + c + d) + b3 (c + d + a) + c3 (d + a + b) + d3 (a + b + c)
Solución.
f (a, b, c, d) = a3 (b + c + d) + b3 (c + d + a) + c3 (d + a + b)
+d3 (a + b + c)
= (a3 b + ab3 ) + (a3 c + ac3 ) + (a3 d + ad3 )
+ (b3 c + bc3 ) + (b3 d + bd3 ) + (c3 d + cd3 )
= ab (a2 + b2 ) + ac (a2 + c2 ) + ad (a2 + d2 )
+bc (b2 + c2 ) + bd (b2 + d2 ) + cd (c2 + d2 )
Pero
luego
f (a, b, c, d)
3 (a2 + b2 + c2 + d2 ) + 6 (a2 b2 + a2 c2 + a2 d2 + b2 c2 + b2 d2 + c2 d2 )
=
4
(a − b)4 + (a − c)4 + (a − d)4 + (b − c)4 + (b − d)4 + (c − d)4
−
4
2
1
z }| {
2 2 2 2
3 a + b + c + d −[(a−b)4 +(a−c)4 +(a−d)4 +(b−c)4 +(b−d)4 +(c−d)4 ]
= 4
3 1
= − [(a − b)4 + (a − c)4 + (a − d)4 + (b − c)4 + (b − d)4 + (c − d)4 ]
4 4
3
≤ .
4
1
La igualdad ocurre cuando a = b = c = d = . 2
2
250 12. Soluciones
a+b+c √
3 √ √ 2 √ √ 2 √ √ 2
− abc ≤ máx a− b , b− c , c− a .
3
a≤b≤c
a+b+c √
3 √ √ 2
− abc ≤ c− a .
3
Definamos la función
a+x+c √ x+a+c √
f (x) = − 3 axc = − 3 acx
3 3
donde a ≤ x ≤ c, luego
1 1 −2/3
f ′ (x) = − x
3 3
2 −5/3 √
f ′′ (x) = x ac ≥ 0, x ≥ 0
9
√ √ 2
f (a) ≤ ( c − a)
1 √3 √
⇔ (2a + c) − a2 c ≤ c + a − 2 ac
3
251
√
3 1 2 √
⇔ a2 c + a + c − 2 ac ≥ 0
3 3
√
3 √
3 a2 c + a + 2c ≥ 6 ac
a+b+c √
3 √ √ 2 √ √ 2 √ √ 2
∴ − abc ≤ máx a− b , b− c , c− a .
3
b+c
Prueba. Si a − ≥ 0, entonces la desigualdad se cumple trivialmente
2
3 3 3
pues a + b + c ≥ 3abc.
b+c
Si a − < 0 definimos
2
3
3 3 3 b+c
f (a, b, c) = a + b + c − 3abc + 2 a −
2
3
3 3 3 2a + 2p + 2q
f = a + (a + p) + (a + q) − 3a(a + p)(a + q) + 2 a −
2
252 12. Soluciones
efectuando
entonces
! !
5
X 5
X
cos xi ≤ | cos x1 | + sen xi
i=1 i=2
entonces
!
5
X
cos xi ≤ | cos x1 | + | cos x2 | + | cos x3 | + | cos x4 | + | cos x5 |
i=1
a2 b2 c2
+ + ≥ 2.
(b − c)2 (c − a)2 (a − b)2
2
a b c
+ +
b−c c−a a−b
2 2 2
a b c
= + +
b−c c−a a−b
ab ac bc
+2 + +
(b − c)(c − a) (b − c)(a − b) (c − a)(a − b)
| {z }
(−1)
2
a b c
⇒ + + +2
b−c c−a a−b
2 2 2
a b c
= + +
b−c c−a a−b
Como
2
a b c
+ + +2≥2
b−c c−a a−b
entonces
a2 b2 c2
+ + ≥ 2.
(b − c)2 (c − a)2 (a − b)2
254 12. Soluciones
a b c
+ + = 0.
b−c c−a a−b
2
4x2
√ 2 < 2x + 9.
1 − 1 + 2x
Solución.
√ 2
4x2 1 + 1 + 2x
√ 2 √ 2 < 2x + 9
1 − 1 + 2x 1 + 1 + 2x
√ 2
4x2 1 + 1 + 2x
⇔ < 2x + 9
(−2x)2
√ 2
4x2 1 + 1 + 2x
⇔ < 2x + 9
4x2
√
⇔ 1 + 2 1 + 2x + 1 + 2x < 2x + 9
255
√
⇔ 2 1 + 2x < 7
⇔ 4(1 + 2x) < 49
⇔ 8x < 45
45
⇔ x< ,
8
45
de donde x ∈ h−∞, i.
8
Luego
45 45
C.S. = h−∞, i ∩ U = [−1/2, i − {0}.
8 8
2
a2 (b + c − a) + b2 (a + c − b) + c2 (a + b − c) ≤ 3abc.
a2 b + a2 c + b2 a + b2 c + c2 a + c2 b ≤ 3abc + a3 + b3 + c3
⇔ 0 ≤ a3 − a2 (b + c) + abc + b3 − b2 (a + c) + abc + c3 − c2 (a + b) + abc
⇔ 0 ≤ a(a − b)(a − c) + b(b − a)(b − c) + c(c − a)(c − b)
⇔ a(a − b)(a − c) + b(b − a)(b − c) + c(c − a)(c − b) ≥ 0,
x1 x2 xn 1 1 1
2
+ 2 +···+ 2 ≥ + +···+ .
1 2 n 1 2 n
x1 x2 xn
+ + · · · + ≥ c1 b1 + c2 b2 + · · · + cn bn
12 22 n2
≥ an b1 + an−1 b2 + · · · + a1 bn
= a1 bn + a2 bn−1 + · · · + an b1
a1 a1 an
= 2 + 2 +···+ 2
1 2 n
como a1 ≥ 1, a2 ≥ 2, . . . , an ≥ n entonces
a1 a1 an 1 2 n
+ + · · · + ≥ + + · · · +
12 22 n2 12 22 n2
1 1 1
= + +···+
1 2 n
257
de donde se obtiene
x1 x2 xn 1 1 1
2
+ 2 +···+ 2 ≥ + +···+ .
1 2 n 1 2 n
2
n+1
|x1 + x2 + · · · + xn | = 1 y |xi | ≤ , para todo i = 1, 2, . . . , n,
2
n+1
|y1 + 2y2 + · · · + nyn | ≤ .
2
⇒ M + N = (n + 1) (x1 + x2 + · · · + xn )
⇒ |M + N| = (n + 1) |x1 + x2 + · · · + xn | = (n + 1) · 1 = n + 1
⇒ |M + N| = n + 1
n+1
i) Si M + N = n + 1 ⇒ N < <M y
2
(n + 1)
ii) Si M + N = −(n + 1) ⇒ N < − < M.
2
Gráficamente
n+1
n+1
N − 2 M N 2 M
258 12. Soluciones
(n + 1) n + 1
Si tomaramos el primer caso, entonces N ∈ − , y en el segundo
2 2
(n + 1) n + 1
caso M ∈ − , , es decir M o N se encuentran en el intervalo
2 2
(n + 1) n + 1 (n + 1) n+1
− , y además N ≤ − < ≤ M.
2 2 2 2
Sean (y1 , y2 , . . . , yn ) una permutación de (x1 , x2 , . . . , xn ) tal que P = 1y1 +
2y2 + · · · + nyn .
(n + 1)
Tomamos el máximo valor de P ≤ − y también tomamos i tal que
2
y1 ≤ y2 ≤ · · · ≤ yn y yi > yi+1 y consideramos
entonces
pruebe que ti , tj , tk son los lados de un triángulo para todo i, j, k con 1 ≤ i <
j < k ≤ n.
X
t1 t2 t1 t3 ti tj
= n+ + + + + +
t2 t1 t3 t1 tj ti
1≤i<j≤n
(i, j) 6= (1, 2)
(i, j) 6= (1, 3)
X
1 1 t2 + t3 ti tj
= n + t1 + + + +
t2 t3 t1 tj ti
1≤i<j≤n
(i, j) 6= (1, 2)
(i, j) 6= (1, 3)
√
2t1 2 t2 t3 n(n − 1)
= n+ √ + +2 −2
t2 t3 t1 2
t1
Haciendo √ = a, tenemos
t2 t3
1 1 1 2
(t1 + t2 + · · · + tn ) + +···+ ≥ n + 2a + + n2 − n − 4
t1 t2 tn a
260 12. Soluciones
como
2 1 1 1
n + 1 > (t1 + t2 + · · · + tn ) + +···+
t1 t2 tn
por transitividad
2
n2 + 1 > n + 2a + + n2 − n − 4
a
2
⇔ 2a + − 5 < 0; a > 0
a
⇔ 2a2 − 5a + 2 < 0
⇔ (2a − 1)(a − 2) < 0
1
⇔ <a<2
2
1 t1
⇔ <√ <2
2 t2 t3
√
⇒ t1 < 2 t2 t3
√
Como 2 t2 t3 ≤ t2 + t3 ,
∴ t1 < t2 + t3 .
119. (IMO 2000). Sean a, b, c números reales positivos tales que abc = 1, pruebe
que
1 1 1
a−1+ b−1+ c−1+ ≤ 1.
b c a
x y z
a= , b = , c = , x, y, z > 0
y z x
x y y z z x
− +1 − +1 − +1 ≤1
z z x x y y
⇔ (x − y + z)(y − z + x)(z − x + y) ≤ xyz (12.22)
Haciendo u = x − y + z, v = y − z + x, w = z − x + y
⇒ u + v = 2x, u + w = 2z, v + w = 2y
u+v v+w u+w
u·v·w ≤
2 2 2
⇔ 8u · v · w ≤ (u + v)(v + w)(u + w) (12.23)
8u · v · w ≤ (u + v)(v + w)(u + w)
se verifica pues
u + v = 2x < 0
u + w = 2z < 0
v + w = 2y < 0
n
X nX
|i − j| |xi − xj | = |xi − xj |
i,j=1
2 i,j
si x1 ≤ x2 ≤ · · · ≤ xn .
Prueba.
n
X X
|i − j| |xi − xj | = |(i − j)(xi − xj )|
i,j=1 i,j
n
Xn X X n Xn
= (i − j)(x i − x )
j = (i − j)(xi − xj );
| {z }
i=1 j=1 i=1 j=1
(+)
(pues x1 ≤ x2 ≤ · · · ≤ xn )
n X
X n
= (ixi − ixj − jxi + jxj )
i=1 j=1
n X
X n n X
X n n X
X n n X
X n
= ixi − ixj − jxi + jxj
i=1 j=1 i=1 j=1 i=1 j=1 i=1 j=1
263
n
X n
X n
X n
X n
X n
X
= nixi − i xj − xi j+ njxj
i=1 i=1 j=1 i=1 j=1 j=1
n
X n
X n
X n
n(n + 1) n(n + 1) X
= nixi − xj − xi · + nixi
i=1
2 j=1 i=1
2 i=1
| {z }
n
X n
X
= 2nixi − n(n + 1) xi
i=1 i=1
Xn
= n (2i − (n + 1)) xi
i=1
efectuando la sumatoria
n
nX X
|xi − xj | = n |xi − xj |
2 i,j=1 i<j
X
=n (xi − xj )
1≤i<j≤n
n
X nX
∴ |i − j| |xi − xj | = |xi − xj |.
i,j=1
2 i,j
2
264 12. Soluciones
n
X n2 (n2 − 1)
(i − j)2 = .
i,j=1
6
Prueba.
n
X n
X
(i − j)2 = i2 − 2ij + j 2
i,j=1 i,j=1
n
X n
X n
X
= i2 − 2 ij + j2
i,j=1 i,j=1 i,j=1
X n
n X n X
X n n X
X n
= i2 − 2 ij + j2
i=1 j=1 i=1 j=1 i=1 j=1
n
X n
X Xn
2 n(n + 1)
= n·i −2 i· +n j2
i=1 i=1
2 j=1
2
265
n
!2 n
X 2 2 X
|xi − xj | ≤ n −1 (xi − xj )2 .
i,j=1
3 i,j=1
!2
X X X
|i − j| |xi − xj | ≤ (i − j)2 · (xi − xj )2
i,j i,j i,j
| {z }
!2
nX n2 (n2 − 1) X
⇔ |xi − xj | ≤ · (xi − xj )2
2 i,j 6 i,j
n2 P 2 n2 (n2 − 1) X
⇔ i,j |xi − xj | ≤ · (xi − xj )2
4 6 i,j
P 2 2 2 X
⇔ i,j |xi − xj | ≤ (n − 1) (xi − xj )2 .
3 i,j
ab(a2 − b2 ) + bc(b2 − c2 ) + ca(c2 − a2 ) ≤ M a2 + b2 + c2 2
ab(a2 − b2 ) + bc(b2 − c2 ) + ca(c2 − a2 ) = |(a − b)(b − c)(c − a)(a + b + c)|
Haciendo
a − b = x, b − c = y, c − a = z ⇒ x + y + z = 0
y además haciendo α = a + b + c
sumando tenemos:
α2 + x2 + y 2 + z 2 = 3 (a2 + b2 + c2 )
2
2 2 2 2 (α2 + x2 + y 2 + z 2 )
⇒ (a + b + c ) = .
9
como
x + y + z = 0 ⇒ −z = x + y
⇒ |z| = |x + y|,
267
entonces
(x + y)2
como xy ≤
4
9
⇒ 9|αxy(x + y)| ≤ |α||x + y|3.
4
9 2
Luego debemos encontrar la relación |α||(x + y)3| ≤ M (α2 + x2 + y 2 + z 2 ) .
4
Aplicando MA ≥ MG:
Pero
(x + y)2 ≤ 2(x2 + y 2)
⇔ 3(x + y)2 ≤ 2(x2 + y 2) + 2 (x + y)2
| {z }
⇔ 3(x + y)2 ≤ 2(x2 + y 2) + 2z 2
3
⇔ (x + y)2 ≤ (x2 + y 2 + z 2 )
2
3
⇔ α2 + (x + y)2 ≤ α2 + x2 + y 2 + z 2
2
2
2 3 2 2
⇔ α + (x + y) ≤ (α2 + x2 + y 2 + z 2 )
2
√ 2 √
9 2 2 3 2 9 2 2 2
⇔ α + (x + y) ≤ (α + x2 + y 2 + z 2 )
32 2 32
√
9 9 2 2 2
⇒ |α||(x + y)3 | ≤ (α + x2 + y 2 + z 2 ) ,
4 32
√
9 2
entonces M ≥
32
√
9 2
de donde el menor valor de M es y esto ocurre si y sólo si (a, b, c) =
32
3 3
1 + √ , 1, 1 − √ . 2
2 2
Prueba. Haciendo
a + b − c = 2y
a = x + y, b = y + z, c = z + x ⇒ a + c − b = 2x
b + c − a = 2z
269
efectuando tenemos:
x2 y 2 z 2 (x + y + z)2
+ + ≥ =x+y+z
y z x y+z+x
de donde
x2 y 2 z 2
+ + ≥ x + y + z.
y z x
2
125. Sean a, b, c, d, e números reales positivos tales que abcde = 1, pruebe que
x y z w t
a= , b= , c= , d= , e=
y z w t x
270 12. Soluciones
en efecto
x x 1 1
+ +
a + abc y w y w
= x x =
1 + ab + abcd 1+ + 1 1 1
z t + +
x z t
1 1 1 1 1
= m, = n, = p, = q, =r
x y z w t
luego
a + abc n+q
= ; similarmente
1 + ab + abcd m+p+r
b + bcd p+r
= ;
1 + bc + bcde n+q+t
c + cde q+m
= ;
1 + cd + cdea p+n+r
d + dea r+n
= ;
1 + de + deab m+p+q
e + eab m+p
= ;
1 + ea + eabc n+q+r
Haciendo
m + p + r = a′ , n + q + t = b′ , p + n + r = c′ ,
m + p + q = d′ , n + q + r = e′ ,
la desigualdad es equivalente a:
′ ′ ′ ′ 1 ′1 1 1 1
(a + b + c + d + e ) + + + + ≥ 25
a′ b′ c′ d′ e′
a′ + b′ + c′ + d′ + e′ 5
⇔ ≥ ,
5 1 1 1 1 1
+ + + +
a′ b′ c′ d′ e′
Prueba. Como
" #2
2 2
3(a b2 + c}2 )
| + {z = [2(a − b)2 + 2(a − c)(b − c) + (a + b + c)2 ]
1
√
⇔ 32 ≥ 16 2 (a − c)(b − c)(a − b)(a + b + c)
| {z }
P
9
⇔ P ≤ √ .
16 2
√
9 2
∴ el máximo de P es . 2
32
x2 x2 x2
+ + ≥1 (12.24)
(x − 1)2 (x − 1)2 (x − 1)2
Prueba.
con abc + ab + ac + bc + a + b + c = 0.
Efectuando tenemos
Además, como
elevando al cuadrado
a2 b2 c2 + (a + b + c)2 + 2abc(a + b + c)
= a2 b2 + a2 c2 + b2 c2 + 2abc(a + b + c)
⇔ a2 b2 c2 + (a + b + c)2 = a2 b2 + a2 c2 + b2 c2 ,
reemplazando en (12.25)
a + b + c − abc = 0
⇔ a + b + c = abc,
2(a + b + c) + ab + ac + bc = 0
⇔ 2(x + y + z − 3) + (x − 1)(y − 1) + (x − 1)(z − 1)
+(y − 1)(z − 1) = 0
⇔ 2(x + y + z) − 6 + xy − (x + y) + 1 + xz − (x + z) + 1
+yz − (y + z) + 1 = 0
⇔ xy + xz + yz = 3
1 1
⇔ xy + + = 3, pues xyz = 1
y x
2 1
⇔ yx + − 3 x + 1 = 0,
y
274 12. Soluciones
2
BIBLIOGRAFÍA
275
276 BIBLIOGRAFÍA